BIO II: Chapters 24-27

Pataasin ang iyong marka sa homework at exams ngayon gamit ang Quizwiz!

Which of the following pairs of protists and their characteristics is mismatched? A) apicomplexans—internal parasites B) golden algae—planktonic producers C) euglenozoans—unicellular flagellates D) ciliates—red tide organisms E) entamoebas—ingestive heterotrophs

ciliates—red tide organisms

Mitochondria are thought to be the descendants of certain alpha proteobacteria. They are, however, no longer able to lead independent lives because most genes originally present on their chromosome have moved to the nuclear genome. Which phenomenon accounts for the movement of these genes? A) plasmolysis B) conjugation C) translation D) endocytosis E) horizontal gene transfer

horizontal gene transfer

Prokaryotes' essential genetic information is located in the A) nucleolus. B) nucleoid. C) nucleosome. D) plasmids. E) exospore.

nucleoid.

Reinforced, threadlike pseudopods that can perform phagocytosis are generally characteristic of which group(s)? A) radiolarians and forams B) gymnamoebas C) entamoebas D) amoeboid stage of cellular slime molds E) oomycetes

radiolarians and forams

Plastids that are surrounded by more than two membranes are evidence of: A) evolution from mitochondria. B) fusion of plastids. C) origin of the plastids from archaea. D) secondary endosymbiosis. E) budding of the plastids from the nuclear envelope.

secondary endosymbiosis.

The last common ancestor of fungi and animals was most likely a: A) colonial photosynthetic eukaryote. B) multicellular photosynthetic eukaryote. C) colonial photosynthetic prokaryote. D) multicellular photosynthetic prokaryote. E) single-celled eukaryote.

single-celled eukaryote

In a hypothetical situation, the genes for sex pilus construction and for tetracycline resistance are located together on the same plasmid within a particular bacterium. If this bacterium readily performs conjugation involving a copy of this plasmid, then the result should be A) a bacterium that has undergone transduction. B) the rapid spread of tetracycline resistance to other bacteria in that habitat. C) the subsequent loss of tetracycline resistance from this bacterium. D) the production of endospores among the bacterium's progeny. E) the temporary possession by this bacterium of a completely diploid genome.

the rapid spread of tetracycline resistance to other bacteria in that habitat.

Nitrogenase, the enzyme that catalyzes nitrogen fixation, is inhibited whenever free O2 reaches a critical concentration. Consequently, nitrogen fixation cannot occur in cells wherein photosynthesis produces free O2. Consider the colonial aquatic cyanobacterium Anabaena, whose heterocytes are described as having "...a thickened cell wall that restricts entry of O2 produced by neighboring cells. Intracellular connections allow heterocysts to transport fixed nitrogen to neighboring cells in exchange for carbohydrates." -Given that the enzymes that catalyze nitrogen fixation are inhibited by oxygen, what are two "strategies" that nitrogen-fixing prokaryotes might use to protect these enzymes from oxygen? 1. couple them with photosystem II (the photosystem that splits water molecules) 2. package them in membranes that are impermeable to all gases 3. be obligate anaerobes 4. be strict aerobes 5. package these enzymes in specialized cells or compartments that inhibit oxygen entry A) 1 and 4 B) 2 and 4 C) 2 and 5 D) 3 and 4 E) 3 and 5 -Which two of the following questions arise from a careful reading of this quotation, and are most important for understanding how N2 enters heterocysts, and how O2 is kept out of heterocysts? 1. If carbohydrates can enter the heterocysts from neighboring cells via the "intracellular connections," how is it that O2 doesn't also enter via this route? 2. If the cell walls of Anabaena's photosynthetic cells are permeable to O2 and CO2, are they also permeable to N2? 3. If the nuclei of the photosynthetic cells contain the genes that code for nitrogen fixation, how can these cells fail to perform nitrogen fixation? 4. If the nuclei of the heterocysts contain the genes that code for photosynthesis, how can these cells fail to perform photosynthesis? 5. If the cell walls of Anabaena's heterocysts are permeable to N2, how is it that N2 doesn't diffuse out of the heterocysts before it can be fixed? 6. If the thick cell walls of the heterocysts exclude entry of oxygen gas, how is it that they don't also exclude the entry of nitrogen gas? A) 1 and 3 B) 1 and 6 C) 2 and 5 D) 3 and 4 E) 4 and 6

-3 and 5 -1 and 6

The following questions are based on the observation that several dozen different proteins comprise the prokaryotic flagellum and its attachment to the prokaryotic cell, producing a highly complex structure. -If the complex protein assemblage of the prokaryotic flagellum arose by the same general processes as those of the complex eyes of molluscs (such as squids and octopi), then A) natural selection cannot account for the rise of the prokaryotic flagellum. B) ancestral versions of this protein assemblage were either less functional or had different functions than modern prokaryotic flagella. C) scientists should accept the conclusion that neither eyes nor flagella could have arisen by evolution. D) we can conclude that both of these structures must have arisen through the direct action of an "intelligent designer." -Certain proteins of the complex motor that drives bacterial flagella are modified versions of proteins that had previously belonged to plasma membrane pumps. This evidence supports the claim that A) some structures are so complex that natural selection cannot, and will not, explain their origins. B) the power of natural selection allows it to act in an almost predictive fashion, producing organs that will be needed in future environments. C) the motors of bacterial flagella were originally synthesized abiotically. D) natural selection can produce new structures by coupling together parts of other structures. E) bacteria that possess flagella must have lost the ability to pump certain chemicals across their plasma membranes.

-ancestral versions of this protein assemblage were either less functional or had different - natural selection can produce new structures by coupling together parts of other structures.

Which two structures play direct roles in permitting bacteria to adhere to each other, or to other surfaces? 1. capsules 2. endospores 3. fimbriae 4. plasmids 5. flagella

1 and 3

Which two genera have members that can evade the human immune system by frequently changing their surface proteins? 1. Plasmodium 2. Trichomonas 3. Paramecium 4. Trypanosoma 5. Entamoeba A) 1 and 2 B) 1 and 4 C) 2 and 3 D) 2 and 4 E) 4 and 5

1 and 4

Paulinella chromatophora is one of the few cercozoans that is autotrophic, carrying out aerobic photosynthesis with its two elongated "cyanelles." The cyanelles are contained within vesicles of the host cell, and each is derived from a cyanobacterium, though not the same type of cyanobacterium that gave rise to the chloroplasts of algae and plants. 1) What must occur for asexual reproduction to be successful in P. chromatophora? 1. mitosis 2. S phase 3. meiosis 4. equal distribution of cyanelles during cytokinesis A) 1 only B) 1 and 2 C) 1, 2, and 3 D) 1, 2, and 4 E) 2, 3, and 4 2) The closest living relative of P. chromatophora is the heterotroph P. ovalis. What type of evidence permits biologists to make this claim about relatedness? A) morphological B) ecological C) biochemical D) genetic E) fossil 3) The closest living relative of P. chromatophora is the heterotroph P. ovalis. P. ovalis uses threadlike pseudopods to capture its prey, which it digests internally. Which of the following, if observed, would be the best reason for relabeling P. chromatophora as a mixotroph? A) a threadlike pseudopod B) a pigmented central vacuole, surrounded by a tonoplast C) a vacuole with food inside D) a secretory vesicle E) a contractile vacuole 4) Which of the following represents the true significance of the finding that the cyanelles of P. chromatophora stem from a different type of cyanobacterium than gave rise to chloroplasts? A) This finding indicates that there is a second evolutionary lineage of photosynthetic eukaryotes. B) This finding represents the first time that primary endosymbiosis has been directly observed. C) This finding is the strongest evidence yet for the theory of endosymbiosis. D) This finding is an example of the phenomenon known as "serial endosymbiosis." E) This finding is the first evidence that eukaryotic cells do not necessarily digest prokaryotic cells that manage to gain access to their cytoplasm. 5) The genome of modern chloroplasts is roughly 50% of the size of the genome of the cyanobacterium from which it is thought to have been derived. In comparison, the genome of P. chromatophora's cyanelle is only slightly reduced relative to the size of the genome of the cyanobacterium from which it is thought to have been derived. What is a valid hypothesis that can be drawn from this comparison? A) Lytic phage infections have targeted the chloroplast genome more often than the P. chromatophora genome. B) P. chromatophora's cyanelle is the result of an evolutionarily recent endosymbiosis. C) The genome of the chloroplast ancestor contained many more introns that could be lost without harm, compared to the cyanelle's genome. D) All three of the hypotheses above are valid. E) Only A and B are correct. 6) If true, which of the following would be most important in determining whether P. chromatophora's cyanelle is still an endosymbiont, or is an organelle, as the term cyanelle implies? A) if P. chromatophora is less fit without its cyanelle than with it B) if the cyanelle is less fit without the host cercozoan than with it C) if there is ongoing metabolic cooperation between the cyanelle and the host cercozoan D) if the magnesium-containing porphyrin ring in the cyanelle's chlorophyll molecules is built by the cyanelle, whereas the organic portion of the chlorophyll molecules is built by the host cercozoan E) if there has been movement of genes from the cyanelle genome to the nuclear genome, such that these genes are no longer present in the cyanelle genome 7) If true, which of the following is the best evidence that the cyanelles are providing nutrition (in other words, calories) to the surrounding cercozoan? A) if the cyanelle performs aerobic photosynthesis B) if the vesicle membrane that surrounds each cyanelle possesses glucose-transport proteins C) if the cyanelle performs aerobic respiration D) if radiolabeled 14CO2 enters the cyanelle and if, subsequently, radiolabeled glucose is present in cercozoan cytosol E) if radiolabeled "heavy" water, 2H2O, enters the cyanelle and if, subsequently, radiolabeled oxygen appears in cercozoan cytosol 8) A crucial photosynthetic gene of the cyanobacterium that gave rise to the cyanelle is called psaE. This gene is present in the nuclear genome of the cercozoan, but is not in the genome of the cyanelle. This is evidence of A) reciprocal mutations in the cyanelle and nuclear genomes. B) horizontal gene transfer from bacterium to eukaryotes. C) genetic recombination involving a protist and an archaean. D) the origin of photosynthesis in protists. E) transduction by a phage that infects both prokaryotes and eukaryotes. 9) Including the membrane of the surrounding vesicle, how many phospholipid (NOT lipopolysaccharide) bilayers should be found around each cyanelle, and which one of these bilayers should have photosystems embedded in it? A) two; innermost B) two; outermost C) three; innermost D) three; middle E) three; outermost

1) 1, 2, and 4 2) genetic 3) a vacuole with food inside 4) This finding indicates that there is a second evolutionary lineage of photosynthetic eukaryotes. 5) P. chromatophora's cyanelle is the result of an evolutionarily recent endosymbiosis. 6) if there has been movement of genes from the cyanelle genome to the nuclear genome, such that these genes are no longer present in the cyanelle genome 7) if radiolabeled 14CO2 enters the cyanelle and if, subsequently, radiolabeled glucose is present in cercozoan cytosol 8) horizontal gene transfer from bacterium to eukaryotes 9) two: innermost

Giardia intestinalis is an intestinal parasite of humans and other mammals that causes intestinal ailments in most people who ingest the cysts. Upon ingestion, each cyst releases two motile cells, called trophozoites. These attach to the small intestine's lining via a ventral adhesive disk. The trophozoites anaerobically metabolize glucose from the host's intestinal contents to produce ATP. Reproduction is completely asexual, occurring by longitudinal binary fission of trophozoites, with each daughter cell receiving two haploid nuclei (n = 5). A trophozoite will often encyst as it passes into the large intestine by secreting around itself a case that is resistant to cold, heat, and dehydration. Infection usually occurs by drinking untreated water that contains cysts. 1) The trophozoites of Giardia were first observed in 1681 in the diarrhea stools of the first known person to view protists with a microscope, a person named A) Robert Koch. B) Robert Hooke. C) Isaac Newton. D) van Leeuwenhoek. E) Louis Pasteur. 2) Giardia's mitosome can be said to be "doubly degenerate," because it is a degenerate form of ________, an organelle that is itself a degenerate form of ________. A) nucleus; archaean B) nucleus; bacterium C) mitochondrion; proteobacterium D) mitochondrion; spirochete E) chloroplast; cyanobacterium . 3) The mitosome of Giardia has no DNA within it. If it did contain DNA, then what predictions should we be able to make about its DNA? 1. It is linear. 2. It is circular. 3. It has many introns. 4. It has few introns. 5. It is not associated with histone proteins. 6. It is complexed with histone proteins. A) 1, 3, and 5 B) 1, 4, and 5 C) 2, 3, and 6 D) 2, 4, and 5 E) 2, 4, and 6 4) Given the putative ancestry of Giardia's mitosome, what should we predict is true of the mitosome? A) It has electron transport systems that use oxygen as the final electron acceptor. B) It has a double membrane. C) It has thylakoids. D) It contains microtubules, arranged in the 9 + 2 pattern. E) It contains 80S (eukaryotic) ribosomes. 5) Given its mode of reproduction and internal structures, which of the following should be expected to occur in Giardia at some stage of its life cycle? 1. separation (segregation) of daughter chromosomes 2. crossing over 3. meiosis A) 1 only B) 3 only C) 1 and 2 D) 1 and 3 E) 2 and 3 6) Unlike most excavates, Giardia trophozoites have no oral groove and are unable to form food vacuoles. Thus, we should expect its nutrition (mostly glucose) to come from A) its mitosomes. B) endosymbiotic cyanobacteria. C) the ventral disk by which it adheres to the intestinal lining. D) osmosis involving aquaporins. E) plasma membrane proteins that are transporters or pumps. 7) During passage through the large intestine, a trophozoite will often secrete a case around itself, forming a cyst. Cysts contain four haploid nuclei. When cysts "hatch" within a new host, two trophozoites are released. Thus, which of the following must happen within the cyst, prior to hatching? 1. meiosis 2. nuclear division 3. S phase 4. binary fission A) 1 only B) 1 and 2 C) 2 and 3 D) 2 and 4 E) 2, 3, and 4 8) The cysts of Giardia are most analogous to the A) mitochondria of ancestral diplomonads. B) nuclei of archaeans. C) endospores of bacteria. D) capsids of viruses. 9) If the mitosomes of Giardia contain no DNA, yet are descendants of what were once freeliving organisms, then where are we likely to find the genes that encode their structures, and what accounts for their current location there? A) plasmids; conjugation B) plasmids; transformation C) nucleus; horizontal gene transfer D) nucleus; S phase 10) The primary treatment for giardiasis (infection with Giardia), as well as for trichomoniasis (infection with Trichomonas vaginalis) and for amoebic dysentery (infection with Entamoeba histolytica), is a drug marketed as Flagyl (generic name is metronidazole). The drug also kills anaerobic gut bacteria. Consequently, which of these are cues that Flagyl's mode of action has nothing to do with attacking or disabling the parasites' flagella, as the drug's name might imply? 1. It would also harm the flagellated lining of the human intestine. 2. Entamoeba possesses pseudopods, not flagella, yet it is killed by Flagyl. 3. Prokaryotic flagella and eukaryotic flagella are radically different from each other and unlikely to be harmed by the same chemical. 4. Not all anaerobic gut bacteria possess flagella, yet it kills these bacteria. A) 1 and 2 B) 1 and 3 C) 2 and 3 D) 1, 2, and 4 E) 2, 3, and 4

1) van Leeuwenhoek 2) mitochondrion; proteobacterium 3) 2, 4, and 5 4) It has a double membrane. 5) 1 only 6) plasma membrane proteins that are transporters or pumps. 7) 2, 3, and 4 8) endospores of bacteria 9) nucleus; horizontal gene transfer 10) 2, 3, and 4

In a bacterium that possesses antibiotic resistance and the potential to persist through very adverse conditions, such as freezing, drying, or high temperatures, DNA should be located within, or be part of, which structure(s)? 1. nucleoid region 2. endospore 3. fimbriae 4. plasmids

1, 2, and 4

The termite gut protist Mixotricha paradoxa has at least two kinds of bacteria attached to its outer surface. One kind is a spirochete that propels its host through the termite gut. A second type of bacteria synthesizes ATP, some of which is used by the spirochetes. The locomotion provided by the spirochetes introduces the ATP-producing bacteria to new food sources. Which of the following terms is (are) applicable to the relationship between the two kinds of bacteria? 1. mutualism 2. parasitism 3. symbiosis 4. metabolic cooperation A) 1 only B) 1 and 2 C) 2 and 3 D) 1, 3, and 4 E) 2, 3, and 4

1, 3, and 4

In Fred Griffith's experiments, harmless R strain pneumococcus became lethal S strain pneumococcus as the result of which of the following? 1. horizontal gene transfer 2. transduction 3. conjugation 4. transformation 5. genetic recombination A) 2 only B) 4 only C) 2 and 5 D) 1, 3, and 5 E) 1, 4, and 5

1, 4, and 5

A hypothetical bacterium swims among human intestinal contents until it finds a suitable location on the intestinal lining. It adheres to the intestinal lining using a feature that also protects it from phagocytes, bacteriophages, and dehydration. Fecal matter from a human in whose intestine this bacterium lives can spread the bacterium, even after being mixed with water and boiled. The bacterium is not susceptible to the penicillin family of antibiotics. It contains no plasmids and relatively little peptidoglycan. -This bacterium's ability to survive in a human who is taking penicillin pills may be due to the presence of which of the following? 1. penicillin-resistance genes 2. a secretory system that removes penicillin from the cell 3. a gram-positive cell wall 4. a gram-negative cell wall 5. an endospore A) 1 or 5 B) 2 or 3 C) 4 or 5 D) 2, 3, or 5 E) 2, 4, or 5 -Adherence to the intestinal lining by this bacterium is due to its possession of A) fimbriae. B) pili. C) a capsule. D) a flagellum. E) a cell wall with an outer lipopolysaccharide membrane. -What should be true of the cell wall of this bacterium? A) Its innermost layer is composed of a phospholipid bilayer. B) After it has been subjected to Gram staining, the cell should remain purple. C) It has an outer membrane of lipopolysaccharide. D) It is mostly composed of a complex, cross-linked polysaccharide. E) Two of the responses above are correct. -Some of the proteins that allow this bacterium to swim are related (in an evolutionary sense) to proteins that A) attach to the single chromosome. B) act as restriction enzymes. C) synthesize peptidoglycan for the cell wall. D) move macromolecules out of the cell. E) comprise its ribosomes. -In which feature(s) should one be able to locate a complete chromosome of this bacterium? 1. nucleolus 2. prophage 3. endospore 4. nucleoid A) 4 only B) 1 and 3 C) 2 and 3 D) 3 and 4 E) 2, 3, and 4 -The cell also lacks F factors and F plasmids. Upon its death, this bacterium should be able to participate in A) conjugation. B) transduction. C) transformation. D) A and B E) B and C F) all of the choices are correct. This bacterium derives nutrition by digesting human intestinal contents (in other words, food). Thus, this bacterium should be an A) aerobic chemoheterotroph. B) aerobic chemoautotroph. C) anaerobic chemoheterotroph. D) anaerobic chemoautotroph. -This bacterium derives nutrition by digesting human intestinal contents (in other words, food). Humans lacking this bacterium have no measurable reproductive advantage or disadvantage relative to humans who harbor this bacterium. Consequently, the bacterium can be properly described as which of the following? 1. symbiont 2. endosymbiont 3. mutualist 4. commensal A) 4 only B) 1 and 2 C) 1 and 4 D) 2 and 3 E) 2 and 4

-2, 4, or 5 -a capsule -It has an outer membrane of lipopolysaccharide. -move macromolecules out of the cell. -3 and 4 -transformation. -anaerobic chemoheterotroph -1 and 4

Healthy individuals of Paramecium bursaria contain photosynthetic algal endosymbionts of the genus Chlorella. When within their hosts, the algae are referred to as zoochlorellae. In aquaria with light coming from only one side, P. bursaria gathers at the well-lit side, whereas other species of Paramecium gather at the opposite side. The zoochlorellae provide their hosts with glucose and oxygen, and P. bursaria provides its zoochlorellae with protection and motility. P. bursaria can lose its zoochlorellae in two ways: (1) if kept in darkness, the algae will die; and (2) if prey items (mostly bacteria) are absent from its habitat, P. bursaria will digest its zoochlorellae. 1) Which term best describes the symbiotic relationship of well-fed P. bursaria to their zoochlorellae? A) mutualistic B) commensal C) parasitic D) predatory E) pathogenic 2) The motility that permits P. bursaria to move toward a light source is provided by A) pseudopods. B) a single flagellum composed of the protein flagellin. C) a single flagellum featuring the 9 + 2 pattern. D) many cilia. E) contractile vacuoles. 3) A P. bursaria cell that has lost its zoochlorellae is said to be aposymbiotic. It might be able to replenish its contingent of zoochlorellae by ingesting them without subsequently digesting them. Which of the following situations would be most favorable to the reestablishment of resident zoochlorellae, assuming compatible Chlorella are present in P. bursaria's habitat? A) abundant light, no bacterial prey B) abundant light, abundant bacterial prey C) no light, no bacterial prey D) no light, abundant bacterial prey 4) A P. bursaria cell that has lost its zoochlorellae is aposymbiotic. If aposymbiotic cells have population growth rates the same as those of healthy, zoochlorella-containing P. bursaria in well-lit environments with plenty of prey items, then such an observation would be consistent with which type of relationship? A) parasitic B) commensalistic C) toxic D) predator—prey E) mutualistic 5) Theoretically, P.bursaria can obtain zoochlorella either vertically (via the asexual reproduction of its mother cell) or horizontally (by ingesting free-living Chlorella from its habitat). Consider a P. bursaria cell containing zoochlorellae, but whose habitat lacks free-living Chlorella. If this cell subsequently undergoes many generations of asexual reproduction, if all of its daughter cells contain roughly the same number of zoochlorellae as it had originally contained, and if the zoochlorellae are all haploid and identical in appearance, then what is true? A) The zoochlorellae also reproduced asexually, at an increasing rate over time. B) The zoochlorellae also reproduced asexually, at a decreasing rate over time. C) The zoochlorellae also reproduced asexually, at a fairly constant rate over time. D) The zoochlorellae reproduced sexually, undergoing heteromorphic alternation of generations. E) The zoochlorellae reproduced sexually, undergoing isomorphic alternation of generations. 6) Which term most accurately describes the nutritional mode of healthy P. bursaria? A) photoautotroph B) photoheterotroph C) chemoheterotroph D) chemoautotroph E) mixotroph

1) mutualistic 2) many cilia. 3) abundant light, abundant bacterial prey 4) commensalistic 5) The zoochlorellae also reproduced asexually, at a fairly constant rate over time. 6) mixotroph

Which of the following is the most likely sequence of events, from earliest to most recent, in the evolution of sponges (organisms with many cells that have different functions)? 1) single-celled eukaryote 2) single-celled prokaryote 3) multicellular eukaryote 4) colonial eukaryote A) 2, 1, 4, 3 B) 2, 1, 3, 4 C) 1, 2, 3, 4 D) 1, 2, 4, 3

2, 1, 4, 3

Which location is nearest to basidiocarps? (figure 26.2) A) A B) B C) C D) D

A) A

Which of these might have been observed in the common ancestor of chondrichthyans and osteichthyans? A) a mineralized, bony skeleton B) opercula C) bony fin rays D) a spiral valve intestine E) a swim bladder

A) a mineralized, bony skeleton

Which of the following are the most abundant and diverse of the extant vertebrates? A) ray-finned fishes B) birds C) amphibians D) nonbird reptiles E) mammals

A) ray-finned fishes

An adult animal that possesses bilateral symmetry is most certainly also A) triploblastic. B) a deuterostome. C) eucoelomate. D) highly cephalized.

A) triploblastic.

Which number represents a megaspore mother cell in Figure 26.1? A) 1 B) 3 C) 5 D) 7 E) 11

B) 3

^^^ In the figure given below, the Deuterostomia clade is most closely related to which two main clades? A) Ctenophora and Cnidaria B) Lophotrochozoa and Ecdysozoa C) Cnidaria and Bilateria D) Platyhelminthes and Rotifera E) Echinodermata and Hemichordata

B) Lophotrochozoa and Ecdysozoa

Which taxon is essentially equivalent to the "embryophytes"? A) Viridiplantae B) Plantae C) Pterophyta D) Bryophyta E) Charophycea

B) Plantae

$$$ If a physoclistus fish removes gas from its swim bladder, this fish's density cannot actually change until that gas arrives at the A) mouth. B) gills. C) skin. D) heart. E) anus.

B) gills.

All fungi are A) symbiotic B) heterotrophic C) flagellated D) pathogenic E) decomposers

B) heterotrophic

In Figure 26.1, the process labeled "6" involves A) nuclear fission. B) mitosis. C) meiosis. D) fertilization. E) binary fission.

B) mitosis.

If all of the animals died in a large region of an ocean, we would expect an increase in the A) number of filter feeders. B) number of cyanobacteria. C) number of decomposers. D) clarity of the ocean waters. E) temperature of the ocean waters.

B) number of cyanobacteria.

$$$ Regarding its position in the water column, the same thing that happens to a shark when it stops swimming also happens to a A) physostomus fish when it gulps air. B) physoclistus fish when it moves gas from the blood into the water. C) physoclistus fish when it moves gas from the swim bladder into the blood. D) physoclistus fish when it moves gas from the blood into the swim bladder.

B) physoclistus fish when it moves gas from the blood into the water.

@@@ In how many of the following ways is Tp unlike the typical animal? 1. Tp is multicellular. 2. Tp lacks muscle and nerve cells. 3. Tp has cilia. 4. Tp has a different place where digestion of food occurs. 5. Tp lacks cell walls. A) only one way B) two ways C) three ways D) four ways E) all five ways

B) two ways

### The taller a Brazil nut tree is, 1. the more valuable it is as a source of lumber. 2. the less useful it is to harpy eagles. 3. the greater its photosynthetic rate relative to neighboring plants. A) 1 only B) 1 and 2 C) 1 and 3 D) 2 and 3

C) 1 and 3

How many chromosomes should be in a megasporangium nucleus? A) 4 B) 8 C) 16 D) 24 E) 32

C) 16

The members of which clade in the phylum Cnidaria occur only as polyps? A) Hydrozoa B) Scyphozoa C) Anthozoa D) Cubozoa

C) Anthozoa

A brachiopod can be distinguished from a bivalve by the presence of A) two hinged shells. B) a digestive system with separate mouth and anus. C) a lophophore. D) suspension feeding. E) a distinct head.

C) a lophophore.

In Figure 26.1, which number represents the mature gametophyte? A) 1 B) 3 C) 5 D) 7 E) 11

D) 7

At which location should one find the lowest concentration of fungal enzymes, assuming that the enzymes do not diffuse far from their source, and that no other fungi are present in this habitat? (figure 26.2) A) A B) B C) C D) D

D) D

Which tree depicts the microsporidians as a sister group of the fungi, rather than as a fungus? A) I B) II C) III D) IV

D) IV

What conclusion is apparent from the data in Table 27.1? A) Land animals have more Hox genes than do those that live in water. B) All bilaterian phyla have had the same degree of expansion in their numbers of Hox genes. C) Acoel flatworms should be expected to contain seven Hox genes. D) The expansion in number of Hox genes throughout vertebrate evolution cannot be explained merely by three duplications of the ancestral vertebrate Hox cluster. E) Extant insects all have seven Hox genes.

D) The expansion in number of Hox genes throughout vertebrate evolution cannot be explained merely by three duplications of the ancestral vertebrate Hox cluster.

What do fungi and arthropods have in common? A) Both groups are commonly coenocytic. B) The haploid state is dominant in both groups. C) Both groups are predominantly heterotrophs that ingest their food. D) The protective coats of both groups are made of chitin. E) Both groups have cell walls.

D) The protective coats of both groups are made of chitin.

Which of the following was not a challenge for survival of the first land plants? A) sources of water B) sperm transfer C) desiccation D) animal predation E) lack of sunlight

D) animal predation

If humans had been present to build log structures during the Carboniferous period (they weren't), which plant types would have been suitable sources of logs? A) whisk ferns and epiphytes B) horsetails and bryophytes C) lycophytes and bryophytes D) ferns, horsetails, and lycophytes E) charophytes, bryophytes, and gymnosperms

D) ferns, horsetails, and lycophytes

Sporophylls can be found in which of the following? A) mosses B) liverworts C) hornworts D) pterophytes E) charophytes

D) pterophytes

In which vertebrates is fertilization exclusively internal? A) chondrichthyans, osteichthyans, and mammals B) amphibians, mammals, and reptiles C) chondrichthyans, osteichthyans, and reptiles D) reptiles and mammals E) reptiles and amphibians

D) reptiles and mammals

In most fungi, karyogamy does not immediately follow plasmogamy, which consequently A) means that sexual reproduction can occur in specialized structures. B) results in multiple diploid nuclei per cell. C) allows fungi to reproduce asexually most of the time. D) results in heterokaryotic or dikaryotic cells. E) is strong support for the claim that fungi are not truly eukaryotic.

D) results in heterokaryotic or dikaryotic cells.

In Figure 26.1, which number represents an embryo? A) 1 B) 3 C) 7 D) 9 E) 11

E) 11

Lichens are symbiotic associations of fungi and A) mosses. B) cyanobacteria. C) green algae. D) A and B E) B and C.

E) B and C.

The oldest fossil eukaryote that can be resolved taxonomically is of: A) a red alga that lived 1.2 billion years ago. B) a red alga that lived 635 million years ago. C) a fungus that lived 2 billion years ago. D) a fungus that lived 550 million years ago. E) an Ediacaran that lived 550 million years ago.

a red alga that lived 1.2 billion years ago.

Although not present in all bacteria, this cell covering often enables cells that possess it to resist the defenses of host organisms, especially their phagocytic cells. A) endospore B) sex pilus C) cell wall D) capsule

capsule

Which of the following is an important source of endotoxin in gram-negative species? A) endospore B) sex pilus C) flagellum D) cell wall E) capsule

cell wall

Multicellular colonies of plant cells adhere to each other primarily by ________, whereas multicellular colonies of animal cells are typically held together by ________. A) fusion of cellular membranes; cell walls B) cell walls; fused cellular membranes C) cell walls; proteins D) interlocking chloroplasts; interlocking mitochondria E) interlocking cellular shapes (like puzzle pieces); large, calcified capsules

cell walls; proteins

The evolution of multicellularity in animals required adaptations that promoted: A) cellular migration and intercellular communication. B) cellular migration and cellular adhesion. C) cellular adhesion and intercellular communication. D) photosynthesis and cellular migration. E) cellular fusion and regeneration.

cellular adhesion and intercellular communication

Which of the following obtain energy by oxidizing inorganic substances to obtain energy that is used, in part, to fix CO2? A) photoautotrophs B) photoheterotrophs C) chemoautotrophs D) chemoheterotrophs that perform decomposition E) parasitic chemoheterotrophs

chemoautotrophs

Which of the following was derived from an ancestral cyanobacterium? A) chloroplast B) mitochondrion C) hydrogenosome D) mitosome E) Two of the responses above are correct.

chloroplast

In a sample of pond water, a new organism is identified with the following characteristics: It consists of 70 cells surrounded by rigid cell walls, which join the cells together. Inside each of these identical cells are mitochondria and chloroplasts. Such an organism would most likely be classified as a: A) colonial photosynthetic eukaryote. B) fungal photosynthetic eukaryote. C) colonial photosynthetic prokaryote. D) multicellular photosynthetic prokaryote. E) colonial eukaryotic fungus.

colonial photosynthetic eukaryote

Plantlike photosynthesis that releases O2 occurs in A) cyanobacteria. B) chlamydias. C) archaea. D) actinomycetes. E) chemoautotrophic bacteria.

cyanobacteria

The chloroplasts of land plants are thought to have been derived according to which evolutionary sequence? A) cyanobacteria → green algae → land plants B) cyanobacteria → green algae → fungi → land plants C) red algae → brown algae → green algae → land plants D) cyanobacteria → red algae → green algae → land plants

cyanobacteria → green algae → land plants

Bacteria perform the following ecological roles. Which role typically does not involve symbiosis? A) skin commensalist B) pathogen C) bioluminescent bacteria in fish D) gut mutualist E) decomposer

decomposer

Broad-spectrum antibiotics inhibit the growth of most intestinal bacteria. Consequently, assuming that nothing is done to counter the reduction of intestinal bacteria, a hospital patient who is receiving broad-spectrum antibiotics is most likely to become A) unable to fix carbon dioxide. B) antibiotic resistant. C) unable to fix nitrogen. D) unable to synthesize peptidoglycan. E) deficient in certain vitamins and nutrients.

deficient in certain vitamins and nutrients.

Which of the following is correctly described as a primary producer? A) oomycete B) kinetoplastid C) apicomplexan D) diatom E) radiolarian

diatom

You are given the task of designing an aquatic protist that is a primary producer. It cannot swim on its own, yet must stay in well-lit surface waters. It must be resistant to physical damage from wave action. It should be most similar to a(n) A) diatom. B) dinoflagellate. C) apicomplexan. D) red alga. E) radiolarian.

diatom

The synthesis of new DNA requires the prior existence of oligonucleotides to serve as primers. On Earth, these primers are small RNA molecules. This latter observation is evidence in support of the hypothesized existence of

earlier genetic systems than those based on DNA

All protists are: A) unicellular. B) eukaryotic. C) symbionts. D) monophyletic. E) mixotrophic.

eukaryotic

Assuming that each of these possesses a cell wall, which prokaryotes should be expected to be most strongly resistant to plasmolysis in hypertonic environments? A) extreme halophiles B) extreme thermophiles C) methanogens D) cyanobacteria E) nitrogen-fixing bacteria that live in root nodules

extreme halophiles

A snail-like, coiled, porous test (shell) of calcium carbonate is characteristic of which group? A) diatoms B) foraminiferans C) radiolarians D) gymnamoebas

foraminiferans

Which of the following steps has not yet been accomplished by scientists studying the origin of life? A) synthesis of small RNA polymers by ribozymes B) abiotic synthesis of polypeptides C) formation of molecular aggregates with selectively permeable membranes D) formation of protocells that use DNA to direct the polymerization of amino acids E) abiotic synthesis of organic molecules

formation of protocells that use DNA to direct the polymerization of amino acids

According to the endosymbiotic theory of the origin of eukaryotic cells, how did mitochondria originate? A) from infoldings of the plasma membrane, coupled with mutations of genes for proteins in energy-transfer reactions B) from engulfed, originally free-living proteobacteria C) by secondary endosymbiosis D) from the nuclear envelope folding outward and forming mitochondrial membranes E) when a protoeukaryote engaged in a symbiotic relationship with a protocell

from engulfed, originally free-living proteobacteria

Carl Woese and collaborators identified two major branches of prokaryotic evolution. What was the basis for dividing prokaryotes into two domains? A) microscopic examination of staining characteristics of the cell wall B) metabolic characteristics such as the production of methane gas C) metabolic characteristics such as chemoautotrophy and photosynthesis D) genetic characteristics such as ribosomal RNA sequences E) ecological characteristics such as the ability to survive in extreme environment

genetic characteristics such as ribosomal RNA sequences

The chloroplasts of all of the following are thought to be derived from ancestral red algae, except those of: A) golden algae. B) diatoms. C) dinoflagellates. D) green algae. E) brown algae.

green algae

A large seaweed that floats freely on the surface of deep bodies of water would be expected to lack which of the following? A) thalli B) bladders C) holdfasts D) gel-forming polysaccharides

holdfasts

If a bacterium regenerates from an endospore that did not possess any of the plasmids that were contained in its original parent cell, the regenerated bacterium will probably also A) lack antibiotic-resistant genes. B) lack a cell wall. C) lack a chromosome. D) lack water in its cytoplasm. E) be unable to survive in its normal environment.

lack antibiotic-resistant genes.

Photoautotrophs use: A) light as an energy source and methane as a carbon source. B) light as an energy source and CO2 as a carbon source. C) N2 as an energy source and CO2 as a carbon source. D) CO2 as both an energy source and a carbon source. E) H2S as an energy source and CO2 as a carbon source.

light as an energy source and CO2 as a carbon source

Genetic variation in bacterial populations cannot result from A) transduction. B) transformation. C) conjugation. D) mutation. E) meiosis.

meiosis

The evolution of complex multicellularity in eukaryotes: A) occurred only once, in the common ancestor of all eukaryotes. B) occurred only once, in the common ancestor of all multicellular eukaryotes. C) occurred only once, in the animal lineage. D) is not documented by the fossil record. E) occurred independently in several different eukaryotic lineages

occurred independently in several different eukaryotic lineages

A biologist discovers an alga that is marine, multicellular, and lives at a depth reached only by blue light. This alga probably belongs to which group? A) red algae B) brown algae C) green algae D) dinoflagellates E) golden algae

red algae

Which group is incorrectly paired with its description? A) rhizarians—morphologically diverse group that includes amoebas with threadlike pseudopodia B) diatoms—important producers in aquatic communities C) red algae—acquired plastids by secondary endosymbiosis D) apicomplexans—parasites with intricate life cycles E) diplomonads—protists with modified mitochondria

red algae—acquired plastids by secondary endosymbiosis

The evolution of multicellularity in animals has primarily occurred by the: A) remodeling of old genes. B) deletion of many old genes. C) production of many new genes. D) deletion of many old chromosomes. E) fusion of genes from many ancestral organisms.

remolding of old genes

Fossilized stromatolites: A) more than 2.8 billion years old have not been discovered. B) formed around deep-sea vents. C) resemble structures formed by bacterial communities that are found today in some shallow marine bays. D) provide evidence that plants moved onto land in the company of fungi around 500 million years ago. E) contain the first undisputed fossils of eukaryotes and date from 1.8 billion years ago.

resemble structures formed by bacterial communities that are found today in some shallow marine bays

Which of these observations gives the most support to the endosymbiotic theory for the origin of eukaryotic cells? A) the existence of structural and molecular differences between the plasma membranes of prokaryotes and the internal membranes of mitochondria and chloroplasts B) the similarity in size between the cytosolic ribosomes of prokaryotes and the ribosomes within mitochondria and chloroplasts C) the size disparity between most prokaryotic cells and most eukaryotic cells D) the observation that some eukaryotic cells lack mitochondria

the similarity in size between the cytosolic ribosomes of prokaryotes and the ribosomes

Jams, jellies, preserves, honey, and other foodstuffs with high sugar content hardly ever become contaminated by bacteria, even when the food containers are left open at room temperature. This is because bacteria that encounter such an environment

undergo death as a result of water loss from the cell.

Which pair of alternatives is highlighted by the life cycle of the cellular slime molds, such as Dictyostelium? A) prokaryotic or eukaryotic B) unicellular or multicellular C) diploid or haploid D) autotroph or heterotroph

unicellular or multicellular

Which of the following traits do archaeans and bacteria share? 1. composition of the cell wall 2. presence of plasma membrane 3. lack of a nuclear envelope 4. identical rRNA sequences A) 1 only B) 3 only C) 1 and 3 D) 2 and 3 E) 2 and 4

2 and 3

Match the numbered terms to the description that follows. Choose all appropriate terms. 1. autotroph 2. heterotroph 3. phototroph 4. chemotroph a prokaryote that obtains both energy and carbon as it decomposes dead organisms A) 1 only B) 4 only C) 1 and 3 D) 2 and 4 E) 1, 3, and 4

2 and 4

Match the numbered terms to the description that follows. Choose all appropriate terms. 1. autotroph 2. heterotroph 3. phototroph 4. chemotroph an organism that obtains both carbon and energy by ingesting prey A) 1 only B) 4 only C) 1 and 3 D) 2 and 4 E) 1, 3, and 4

2 and 4

A fish that has been salt-cured subsequently develops a reddish color. You suspect that the fish has been contaminated by the extreme halophile Halobacterium. Which of these features of cells removed from the surface of the fish, if confirmed, would support your suspicion? 1. the presence of the same photosynthetic pigments found in cyanobacteria 2. cell walls that lack peptidoglycan 3. cells that are isotonic to conditions on the surface of the fish 4. cells unable to survive salt concentrations lower than 9% 5. the presence of very large numbers of ion pumps in its plasma membrane A) 2 and 5 B) 3 and 4 C) 1, 4, and 5 D) 3, 4, and 5 E) 2, 3, 4, and 5

2, 3, 4, and 5

In a hypothetical situation, a bacterium lives on the surface of a leaf, where it obtains nutrition from the leaf's nonliving, waxy covering while inhibiting the growth of other microbes that are plant pathogens. If this bacterium gains access to the inside of a leaf, however, it causes a fatal disease in the plant. Once the plant dies, the bacterium and its offspring decompose the plant. What is the correct sequence of ecological roles played by the bacterium in the situation described here? Use only those that apply. 1. nutrient recycler 2. mutualist 3. commensal 4. pathogen 5. primary producer A) 1, 3, 4 B) 2, 3, 4 C) 2, 4, 1 D) 1, 2, 5 E) 1, 2, 3

2, 4, 1

The thermoacidophile Sulfolobus acidocaldarius lacks peptidoglycan, but still possesses a cell wall. What is likely to be true of this species? 1. It is a bacterium. 2. It is an archaean. 3. The optimal pH of its enzymes will lie above pH 7. 4. The optimal pH of its enzymes will lie below pH 7. 5. It could inhabit certain hydrothermal springs. 6. It could inhabit alkaline hot springs. A) 1, 3, and 6 B) 2, 4, and 6 C) 2, 4, and 5 D) 1, 3, and 5 E) 1, 4, and 5

2, 4, and 5

You are given the task of designing an aerobic, mixotrophic protist that can perform photosynthesis in fairly deep water (for example, 250 m deep), and can also crawl about and engulf small particles. With which two of the following structures would you provide your protist? 1. hydrogenosome 2. apicoplast 3. pseudopods 4. chloroplast from red alga 5. chloroplast from green alga A) 1 and 2 B) 2 and 3 C) 2 and 4 D) 3 and 4 E) 4 and 5

3 and 4

Match the numbered terms to the description that follows. Choose all appropriate terms. 1. autotroph 2. heterotroph 3. phototroph 4. chemotroph an organism that obtains energy from light A) 1 only B) 3 only C) 1 and 3 D) 2 and 4 E) 1, 3, and 4

3 only

Approximately how far back in time does the fossil record extend?

3.5 billion years

The predatory bacterium Bdellovibrio bacteriophorus drills into a prey bacterium and, once inside, digests it. In an attack upon a gram-negative bacterium that has a slimy cell covering, what is the correct sequence of structures penetrated by B. bacteriophorus on its way to the prey's cytoplasm? 1. membrane composed mostly of lipopolysaccharide 2. membrane composed mostly of phospholipids 3. peptidoglycan 4. capsule

4, 1, 3, 2

What is thought to be the correct sequence of these events, from earliest to most recent, in the evolution of life on Earth? 1. origin of mitochondria 2. origin of multicellular eukaryotes 3. origin of chloroplasts 4. origin of cyanobacteria 5. origin of fungal-plant symbioses A) 4, 3, 2, 1, 5 B) 4, 1, 2, 3, 5 C) 4, 1, 3, 2, 5 D) 4, 3, 1, 5, 2 E) 4, 3, 1, 2, 5

4, 1, 3, 2, 5

If the kingdom Plantae is someday expanded to include the charophytes, then the shared derived characteristics of the kingdom will include 1. rings of cellulose-synthesizing complexes. 2. chlorophylls a and b. 3. alternation of generations. 4. cell walls of cellulose. 5. ability to synthesize sporopollenin. A) 1 and 5 B) 1, 2, and 3 C) 1, 3, and 5 D) 1, 4, and 5 E) 1, 2, 4, and 5

A) 1 and 5

What is thought to be the correct sequence of the following events during the Carboniferous period? 1. vascular plants become more prevalent 2. megaphylls with large surface areas become more prevalent 3. atmospheric CO2 levels decline by a factor of five 4. global cooling occurs, leading to widespread glaciations A) 1, 2 ,3, 4 B) 2, 1, 3, 4 C) 2, 1, 4, 3 D) 1, 2, 4, 3 E) 3, 4, 1, 2

A) 1, 2 ,3, 4

Arrange the following structures, which can be found on male pine trees, from the largest structure to the smallest structure (or from most inclusive to least inclusive). 1. sporophyte 2. microspores 3. microsporangia 4. pollen cone 5. pollen nuclei A) 1, 4, 3, 2, 5 B) 1, 4, 2, 3, 5 C) 1, 2, 3, 5, 4 D) 4, 1, 2, 3, 5 E) 4, 3, 2, 5, 1

A) 1, 4, 3, 2, 5

If a single, diploid G2 nucleus in an ascus contains 400 nanograms (ng) of DNA, then a single ascospore nucleus of this species should contain how much DNA (ng), carried on how many chromosomes? A) 100, carried on 7 chromosomes B) 100, carried on 14 chromosomes C) 200, carried on 7 chromosomes D) 200, carried on 14 chromosomes E) 400, carried on 14 chromosomes

A) 100, carried on 7 chromosomes

During chordate evolution, what is the sequence (from earliest to most recent) in which the following structures arose? 1. amniotic egg 2. paired fins 3. jaws 4. swim bladder 5. four-chambered heart A) 2, 3, 4, 1, 5 B) 3, 2, 4, 1, 5 C) 3, 2, 1, 4, 5 D) 2, 1, 4, 3, 5 E) 2, 4, 3, 1, 5

A) 2, 3, 4, 1, 5

At which location is the mycelium currently absorbing the most nutrients per unit surface area, per unit time? (figure 26.2) A) A B) B C) C D) D

A) A

Which of the following is a diploblastic phylum of aquatic predators? A) Cnidaria B) Annelida C) Mollusca D) Arthropoda E) Echinodermata

A) Cnidaria

Which of the following statements correctly describes a portion of the pine life cycle? A) Female gametophytes use mitosis to produce eggs. B) Seeds are produced in pollen-producing cones. C) Pollen grains contain female gametophytes. D) A pollen tube slowly digests its way through the triploid endosperm.

A) Female gametophytes use mitosis to produce eggs.

At least one of these has been found in all species of eumetazoan animals studied thus far. A) Hox B) Dlx C) Otx D) FOXP2 E) more than one of these

A) Hox

*** Which phylogeny has been created by emphasizing the structural simplicity of placozoans? A) I B) II C) III

A) I

Which tree depicts the microsporidians as a sister group of the ascomycetes? (look at phylogenic trees) A) I B) II C) III D) IV

A) I

Which of the following is true concerning flowering plants? A) The flower includes sporophyte tissue. B) The gametophyte generation is dominant. C) The gametophyte generation is what we see when looking at a large plant. D) The sporophyte generation is not photosynthetic. E) The sporophyte generation consists of relatively few cells within the flower.

A) The flower includes sporophyte tissue.

A fruit is usually A) a mature ovary. B) a thickened style. C) an enlarged ovule. D) a modified root. E) a mature female gametophyte.

A) a mature ovary.

Which of the following could be considered the most recent common ancestor of living tetrapods? A) a sturdy-finned, shallow-water lobe-fin whose appendages had skeletal supports similar to those of terrestrial vertebrates B) an armored gnathostome with two pairs of appendages C) an early ray-finned fish that developed bony skeletal supports in its paired fins D) a salamander that had legs supported by a bony skeleton but moved with the side-to-side bending typical of fishes E) an early terrestrial caecilian whose legless condition had evolved secondarily

A) a sturdy-finned, shallow-water lobe-fin whose appendages had skeletal supports similar to those of terrestrial vertebrates

###People who attempted to plant Brazil nuts in hopes of establishing plantations of Brazil nut trees played roles most similar to those of A) agoutis. B) orchid bees. C) pollen tubes. D) harpy eagles.

A) agoutis.

###Orchid bees are to Brazil nut trees as ________ are to pine trees. A) breezes B) rain droplets C) seed-eating birds D) squirrels E) both seed-eating birds and squirrels

A) breezes

Which group is noted for the independence of gametophyte and sporophyte generations from each other? A) ferns B) mosses, hornworts, and liverworts C) charophytes D) angiosperms E) gymnosperms

A) ferns

@@@What feature of both true mosses and ferns makes it most surprising that they can survive for many generations in dry deserts? A) flagellated sperm B) lack of vascular tissues C) lack of true roots D) lack of cuticle E) a gametophyte generation that is dominant

A) flagellated sperm

What is the ploidy of a single mature ascospore? A) haploid B) diploid C) triploid D) tetraploid E) polyploid

A) haploid

!!! Which of these features is least useful in assigning the phylum Cycliophora to a clade of animals? A) having a true coelom as a body cavity B) having a body symmetry that permits a U-shaped intestine C) having embryos with spiral cleavage D) lacking ecdysis (molting)

A) having a true coelom as a body cavity

Organisms at which of the following trophic levels increased the most because of the movement of animals onto land? A) herbivores and carnivores B) decomposers and producers C) producers D) decomposers

A) herbivores and carnivores

Evidence indicates that an ancestral finch species from South America arrived on the Galapágos Islands and formed many new species, adapting to the diverse environments on the islands. With the evolution of these new bird species on the Galapágos Islands, we expect to find a corresponding A) increase in the number of bird parasites on the Galapágos Islands. B) decrease in the number of bird parasites on the Galapágos Islands. C) increase in the number of bird parasites in South America. D) decrease in the number of bird parasites in South America. E) elimination of bird parasites on the Galapágos Islands.

A) increase in the number of bird parasites on the Galapágos Islands.

Mitotic activity by the apical meristem of a root makes which of the following more possible? A) increase of the aboveground stem. B) decreased absorption of mineral nutrients. C) increased absorption of CO2. D) increased number of chloroplasts in roots. E) effective lateral growth of the stem.

A) increase of the aboveground stem.

Which extant chordates are postulated to be most like the earliest chordates in appearance? A) lancelets B) adult tunicates C) amphibians D) reptiles E) chondrichthyans

A) lancelets

***A dissection of the interior of this organism's stem should reveal A) lignified vascular tissues. B) cuticle. C) gametangia. D) that it is composed of only a single, long cell. E) a relatively high proportion of dead, water-filled cells.

A) lignified vascular tissues.

$$$ In coelacanths, a swim bladder is present, but the swim bladder is full of adipose tissue (fat), which is there on a fairly permanent basis. If such a swim bladder is used by coelacanths to affect buoyancy, then it does so in much the same way as does the A) liver of a shark. B) physoclistus swim bladder. C) physostomus swim bladder. D) lung of a lungfish.

A) liver of a shark.

Which of the following sex and generation combinations most directly produces the pollen tube? A) male gametophyte B) female gametophyte C) male sporophyte D) female sporophyte

A) male gametophyte

According to the evidence collected so far, the animal kingdom is A) monophyletic. B) paraphyletic. C) polyphyletic. D) euphyletic. E) multiphyletic.

A) monophyletic.

@@@ Tp's body symmetry seems to be most like that of A) most sponges. B) cnidarians. C) worms. D) tetrapods. E) Two of the responses above are correct.

A) most sponges.

Which of the following flower parts develops into a seed? A) ovule B) ovary C) fruit D) stamen Answer: A

A) ovule

Which of the following characteristics is unique to chytrids compared to other groups of fungi? A) presence of flagella B) zoospores C) autotrophic mode of nutrition D) cell walls of cellulose E) nucleotide sequences of several genes

A) presence of flagella

The following are all adaptations in plants to life on land except A) rings of cellulose-synthesizing complexes. B) cuticles. C) tracheids. D) reduced gametophyte generation. E) multicellular, dependent embryos.

A) rings of cellulose-synthesizing complexes.

$$$ If the beetles survive by consuming cycad pollen, then whether the beetles should be considered mutualists with, or parasites of, the cycads depends upon A) the extent to which their overall activities affect cycad reproduction. B) the extent to which the beetles are affected by the neurotoxins. C) the extent to which the beetles damage the cycad flowers. D) the distance the beetles must travel between cycad microsporophylls and cycad megasporophylls.

A) the extent to which their overall activities affect cycad reproduction.

A researcher took water in which a Jl population had been thriving, filtered the water to remove all bacterial cells, and then applied the water to the skins of adult amphibians to see if there would subsequently be a reduced infection rate by Bd when frog skins were inoculated with Bd. For which of the following hypotheses is the procedure described a potential test? A) the hypothesis that a toxin secreted by Jl cells kills Bd cells when both are present together on frog skin B) the hypothesis that Jl cells infect and kill Bd cells when both are present together on frog skin C) the hypothesis that Jl outcompetes Bd when both are present together on a frog's skin D) the hypothesis that the presence of Jl on frog skin causes a skin reaction that prevents attachment by Bd cells

A) the hypothesis that a toxin secreted by Jl cells kills Bd cells when both are present together on frog skin

+++ Which of the following is a point of conflict between the phylogenetic analyses presented in these two figures? A) the relationship of taxa of segmented animals to taxa of nonsegmented animals B) that sponges are basal animals C) that chordates are deuterostomes D) the monophyly of the bilaterians E) the monophyly of the animal kingdom

A) the relationship of taxa of segmented animals to taxa of nonsegmented animals

Which of these characteristics added most to vertebrate success in relatively dry environments? A) the shelled, amniotic egg B) the ability to maintain a constant body temperature C) two pairs of appendages D) bony scales E) a four-chambered heart

A) the shelled, amniotic egg

Which of the following statements concerning protists is true? A) All protists have mitochondria, although in some species they are much reduced and known by different names. B) The primary organism that transmits malaria to humans by its bite is the tsetse fly. C) All apicomplexans are autotrophic. D) All slime molds have an amoeboid stage that may be followed by a stage during which spores are produced. E) Euglenozoans that are mixotrophic lack functional chloroplasts.

All protists have mitochondria, although in some species they are much reduced and known by different names.

If archaeans are more closely related to eukaryotes than to bacteria, then which of the following is a reasonable prediction? A) Archaean DNA should have no introns. B) Archaean chromosomes should have no protein bonded to them. C) Archaean DNA should be single-stranded. D) Archaean ribosomes should be larger than typical prokaryotic ribosomes. E) Archaeans should lack cell walls.

Archaean ribosomes should be larger than typical prokaryotic ribosomes

Which of these, if true, would support the claim that the ancestral cnidarians had bilateral symmetry? 1. Cnidarian larvae possess anterior-posterior, left-right, and dorsal-ventral aspects. 2. Cnidarians have fewer Hox genes than bilaterians. 3. All extant cnidarians, including Nematostella, are diploblastic. 4. β-catenin turns out to be essential for gastrulation in all animals in which it occurs. 5. All cnidarians are acoelomate. A) 1 only B) 1 and 4 C) 2 and 3 D) 2 and 4 E) 4 and 5

B) 1 and 4

If Bd cannot grow properly at temperatures above 28°C (82°F), then, assuming the amphibians can survive, in which time or place should the chytrid infection proceed most rapidly? 1. cooler months 2. warmer months 3. lower altitudes 4. higher altitudes A) 1 or 3 B) 1 or 4 C) 2 or 3 D) 2 or 4

B) 1 or 4

Evidence indicates that plants increase the number of stomata in their leaves as atmospheric CO2 levels decline. Increasing the number of stomata per unit surface area should have the effect of doing which of the following? 1. increasing dehydration of leaf tissues 2. decreasing dehydration of leaf tissues 3. countering the effect of declining CO2 on photosynthesis 4. reinforcing the effect of declining CO2 on photosynthesis 5. decreasing the O2 content of air next to the leaves lower than it would otherwise be 6. increasing the O2 content of air next to the leaves higher than it would otherwise be A) 1, 3, and 5 B) 1, 3, and 6 C) 1, 4, and 5 D) 2, 3, and 6 E) 2, 4, and 5

B) 1, 3, and 6

What is true of the phylogenetic tree in Figure 26.3? 1. It depicts uncertainty about whether the bryophytes or the vascular plants evolved first. 2. It is hypothetical. 3. It includes polytomies. 4. It shows that ferns (phylum Pterophyta) are the closest living relatives to the seed plants. 5. It indicates that seeds are a shared ancestral character of all vascular plants. A) 1 and 2 B) 2 and 3 C) 1, 2, and 3 D) 1, 2, and 4 E) 2, 3, and 5

B) 2 and 3

Arrange the following terms from most inclusive to least inclusive. 1. embryophytes 2. green plants 3. seedless vascular plants 4. ferns 5. tracheophytes A) 1, 2, 5, 3, 4 B) 2, 1, 5, 3, 4 C) 2, 5, 1, 3, 4 D) 1, 4, 2, 5, 3 E) 2, 1, 5, 4, 3

B) 2, 1, 5, 3, 4

Meiosis is most likely to be represented by which number(s) in Figure 26.1? A) 2 B) 4 C) 2 and 8 D) 4 and 8 E) 10 and 12

B) 4

Within a gymnosperm megasporangium, what is the correct sequence in which the following should appear during development, assuming that fertilization occurs? 1. sporophyte embryo 2. female gametophyte 3. egg cell 4. megaspore A) 4 → 3 → 2 → 1 B) 4 → 2 → 3 → 1 C) 4 → 1 → 2 → 3 D) 1 → 4 → 3 → 2 E) 1 → 4 → 2 → 3

B) 4 → 2 → 3 → 1

What is the probable sequence in which the following clades of animals originated, from earliest to most recent? 1. tetrapods 2. vertebrates 3. deuterostomes 4. amniotes 5. bilaterians A) 5 → 3 → 2 → 4 → 1 B) 5 → 3 → 2 → 1 → 4 C) 5 → 3 → 4 → 2 → 1 D) 3 → 5 → 4 → 2 → 1 E) 3 → 5 → 2 → 1 → 4

B) 5 → 3 → 2 → 1 → 4

How many chromosomes should be in a tube cell nucleus? A) 4 B) 8 C) 16 D) 24 E) 32

B) 8

Which of the following is a true statement about plant reproduction? A) Embryophytes are small because they are in an early developmental stage. B) Both male and female bryophytes produce gametangia. C) Gametangia protect gametes from excess water. D) Eggs and sperm of bryophytes swim toward one another. E) Bryophytes are limited to asexual reproduction.

B) Both male and female bryophytes produce gametangia.

In which of the following taxa does the mature sporophyte depend completely on the gametophyte for nutrition? A) Pterophyta B) Bryophyta C) horsetail (Equisetum) D) Pterophyta, Bryophyta, and horsetail (Equisetum) E) Pterophyta and Bryophyta

B) Bryophyta

*** Which phylogeny has been created by emphasizing a protein found in placozoans? A) I B) II C) III

B) II

Why is the amniotic egg considered an important evolutionary breakthrough? A) It has a shell that increases gas exchange. B) It allows deposition of eggs in a terrestrial environment. C) It prolongs embryonic development. D) It provides insulation to conserve heat. E) It permits internal fertilization to be replaced by external fertilization.

B) It allows deposition of eggs in a terrestrial environment.

Which clade in the phylum Cnidaria includes "jellies" with rounded (as opposed to boxlike) medusae? A) Hydrozoa B) Scyphozoa C) Anthozoa D) Cubozoa

B) Scyphozoa

If the current molecular evidence regarding animal origins is well substantiated in the future, then what will be true of any contrary evidence regarding the origin of animals derived from the fossil record? A) The contrary fossil evidence will be seen as a hoax. B) The fossil evidence will be understood to have been interpreted incorrectly because it is incomplete. C) The fossil record will henceforth be ignored. D) Phylogenies involving even the smallest bit of fossil evidence will need to be discarded. E) Only phylogenies based solely on fossil evidence will need to be discarded.

B) The fossil evidence will be understood to have been interpreted incorrectly because it is incomplete.

+++ In the traditional phylogeny (A), the phylum Platyhelminthes is depicted as a sister taxon to the rest of the protostome phyla, and as having diverged earlier from the lineage that led to the rest of the protostomes. In the molecular phylogeny (B), Platyhelminthes is depicted as a lophotrochozoan phylum. What probably led to this change? A) Platyhelminthes ceased to be recognized as true protostomes. B) The removal of the acoel flatworms (Acoela) from the Platyhelminthes allowed the remaining flatworms to be clearly tied to the Lophotrochozoa. C) All Platyhelminthes must have a well-developed lophophore as their feeding apparatus. D) Platyhelminthes' close genetic ties to the arthropods became clear as their Hox gene sequences were studied.

B) The removal of the acoel flatworms (Acoela) from the Platyhelminthes allowed the remaining flatworms to be clearly tied to the Lophotrochozoa.

+++ What is true of the deuterostomes in the molecular phylogeny (B) that is not true in the traditional phylogeny (A)? A) Deuterostomia is paraphyletic. B) To maintain Deuterostomia as a clade, some phyla had to be removed from it. C) Deuterostomia now includes the Acoela. D) It is actually a grade, rather than a clade. E) It diverged from the rest of the Bilateria earlier than did the Acoela.

B) To maintain Deuterostomia as a clade, some phyla had to be removed from it.

Which of the following is true of seedless vascular plants? A) Extant seedless vascular plants are larger than the extinct varieties. B) Whole forests were dominated by large, seedless vascular plants during the Carboniferous period. C) They produce many spores, which are really the same as seeds. D) The gametophyte is the dominant generation. E) Sphagnum is an economically and ecologically important example.

B) Whole forests were dominated by large, seedless vascular plants during the Carboniferous period.

How have fruits contributed to the success of angiosperms? A) by nourishing the plants that make them B) by facilitating dispersal of seeds C) by attracting insects to the pollen inside D) by producing sperm and eggs inside a protective coat E) by producing triploid cells via double fertilization

B) by facilitating dispersal of seeds

Apart from direct amphibian-to-amphibian contact, what is the most likely means by which the zoospores spread from one free-living amphibian to another? A) by wind-blown spores B) by flagella C) by cilia D) by pseudopods E) by hyphae

B) by flagella

In terms of food capture, which sponge cell is most similar to the cnidocyte of a cnidarian? A) amoebocyte B) choanocyte C) epidermal cell D) pore cell

B) choanocyte

Jaws first occurred in which extant group of fishes? A) lampreys B) chondrichthyans C) ray-finned fishes D) lungfishes E) placoderms

B) chondrichthyans

If all of their nuclei are equally active transcriptionally, then the cells of dikaryotic fungi, in terms of the gene products they can make, are essentially A) haploid. B) diploid. C) alloploid. D) completely homozygous. E) completely hemizygous.

B) diploid.

Which of the following is a characteristic of all angiosperms? A) complete reliance on wind as the pollinating agent B) double internal fertilization C) free-living gametophytes D) carpels that contain microsporangia E) ovules that are not contained within ovaries

B) double internal fertilization

Which of the following should have had gene sequences most similar to the charophyte that was the common ancestor of the land plants? A) early angiosperms B) early bryophytes C) early gymnosperms D) early lycophytes E) early pterophytes

B) early bryophytes

$$$ We should expect the inner wall of the swim bladder to be lined with tissue that is derived from A) ectoderm. B) endoderm. C) mesoderm. D) mesoglea. E) neurectoderm.

B) endoderm.

The earliest known mineralized structures in vertebrates are associated with which function? A) reproduction B) feeding C) locomotion D) defense E) respiration

B) feeding

Evidence of which structure or characteristic would be most surprising to find among fossils of the Ediacaran fauna? A) true tissues B) hard parts C) bilateral symmetry D) cephalization E) embryos

B) hard parts

!!! Which of these, if discovered among cycliophorans, would cause the most confusion concerning our current understanding of cycliophoran taxonomy? A) if the ciliated feeding ring is a lophophore B) if embryos are diploblastic C) if the body cavity is actually a pseudocoelom D) if the organisms show little apparent cephalization

B) if embryos are diploblastic

Generally, wind pollination is most likely to be found in seed plants that grow A) close to the ground. B) in dense, single-species stands. C) in relative isolation from other members of the same species. D) along coastlines where prevailing winds blow from the land out to sea. E) in well-drained soils.

B) in dense, single-species stands.

###The harpy eagle, Harpia harpyja, is the largest, most powerful raptor in the Americas. It nests only in trees taller than 25 meters. It is a "sloth specialist," but will also take agouti. Thus, if these eagles capture too many agoutis from a particular locale, they might contribute to their own demise by A) having too many offspring. B) increasing habitat loss. C) decreasing atmospheric CO2. D) increasing the number of sloths.

B) increasing habitat loss.

Both animals and fungi are heterotrophic. What distinguishes animal heterotrophy from fungal heterotrophy is that only animals derive their nutrition by A) preying on animals. B) ingesting it. C) consuming living, rather than dead, prey. D) using enzymes to digest their food.

B) ingesting it.

If haustoria from the fungal partner were to appear within the photosynthetic partner of a lichen, and if the growth rate of the photosynthetic partner consequently slowed substantially, then this would support the claim that A) algae and cyanobacteria are autotrophic. B) lichens are not purely mutualistic relationships. C) algae require maximal contact with the fungal partner in order to grow at optimal rates. D) fungi get all of the nutrition they need via the "leakiness" of photosynthetic partners. E) soredia are asexual reproductive structures combining both the fungal and photosynthetic partners.

B) lichens are not purely mutualistic relationships.

All of the following are common to both charophytes and vascular land plants except A) sporopollenin. B) lignin. C) chlorophyll a. D) cellulose. E) chlorophyll b.

B) lignin.

Which group's members have had both lungs and gills during their adult lives? A) sharks, skates, and rays B) lungfishes C) lancelets D) amphibians E) ichthyosaurs and plesiosaurs

B) lungfishes

If a multicellular animal lacks true tissues, then it can properly be included among the A) eumetazoans. B) metazoans. C) choanoflagellates. D) lophotrochozoans. E) bilateria.

B) metazoans.

The generative cell of male angiosperm gametophytes is haploid. This cell divides to produce two haploid sperm cells. What type of cell division does the generative cell undergo to produce these sperm cells? A) binary fission B) mitosis C) meiosis D) mitosis without subsequent cytokinesis E) meiosis without subsequent cytokinesis

B) mitosis

A new species of aquatic chordate is discovered that closely resembles an ancient form. It has the following characteristics: external armor of bony plates, no paired lateral fins, and a suspension-feeding mode of nutrition. In addition to these, it will probably have which of the following characteristics? A) legs B) no jaws C) an amniotic egg D) endothermy

B) no jaws

Match the extant vertebrate group with the description. Internal fertilization, leathery amniotic egg, and skin that resists drying are characteristics of A) amphibians. B) nonbird reptiles. C) chondrichthyans. D) mammals. E) birds. Answer:

B) nonbird reptiles.

Some green algae exhibit alternation of generations. All land plants exhibit alternation of generations. No charophytes exhibit alternation of generations. Keeping in mind the recent evidence from molecular systematics, the correct interpretation of these observations is that A) charophytes are not related to either green algae or land plants. B) plants evolved alternation of generations independently of green algae. C) alternation of generations cannot be beneficial to charophytes. D) land plants evolved directly from the green algae that perform alternation of generations. E) scientists have no evidence to indicate whether or not land plants evolved from any kind of alga.

B) plants evolved alternation of generations independently of green algae.

&&& The internal fertilization that occurs prior to shell deposition is what type of angiosperm analog? A) endosperm B) pollen tube and sperm nuclei C) carpels D) fruit E) integuments

B) pollen tube and sperm nuclei

Which of the following are structures of angiosperm gametophytes? A) immature ovules B) pollen tubes C) ovaries D) stamens E) sepals

B) pollen tubes

The functional role of sporopollenin is primarily to A) comprise spore surface structures that catch the wind and assist in spore dispersal. B) reduce dehydration. C) make spores less dense and able to disperse more readily. D) repel toxic chemicals. E) provide nutrients to spores.

B) reduce dehydration.

Chordate pharyngeal slits appear to have functioned first as A) the digestive system's opening. B) suspension-feeding devices. C) components of the jaw. D) gill slits for respiration. E) portions of the inner ear.

B) suspension-feeding devices.

As a result of harvesting a higher proportion of older and larger cod and other fish species for food A) the number of fish in these populations has increased dramatically. B) the age and size of fish reaching sexual maturity have decreased. C) many new species of predators have evolved in these regions. D) the affected fish species have stopped reproducing. E) many of the fish in these populations have started to reproduce asexually.

B) the age and size of fish reaching sexual maturity have decreased.

Which of the following clades contains the greatest number of animal species? A) the vertebrates B) the bilaterians C) the sponges D) the deuterostomes E) the insects

B) the bilaterians

The last common ancestor of all bilaterians is thought to have had four Hox genes. Most extant cnidarians have two Hox genes, except Nematostella (of β-catenin fame), which has three Hox genes. On the basis of these observations, some have proposed that the ancestral cnidarians were originally bilateral and, in stages, lost Hox genes from their genomes. If true, this would mean that A) Radiata should be a true clade. B) the radial symmetry of extant cnidarians is secondarily derived, rather than being an ancestral trait. C) Hox genes play little actual role in coding for an animal's "body plan." D) Cnidaria may someday replace porifera as the basal bilaterians. E) cnidarians are the basal metazoans.

B) the radial symmetry of extant cnidarians is secondarily derived, rather than being an ancestral trait.

You are hiking in a forest and come upon a mysterious plant, which you determine is either a lycophyte sporophyte or a pterophyte sporophyte. Which of the following would be most helpful in determining the correct classification of the plant? A) whether or not it has true leaves B) whether it has microphylls or megaphylls C) whether or not it has seeds D) its height E) whether or not it has chlorophyll a

B) whether it has microphylls or megaphylls

###The same bees that pollinate the flowers of the Brazil nut trees pollinate orchids, which are epiphytes (in other words, plants that grow on other plants); however, orchids cannot grow on Brazil nut trees. These observations explain A) the coevolution of Brazil nut trees and orchids. B) why Brazil nut trees do not set fruit in monoculture plantations. C) why male orchid bees do not pollinate Brazil nut tree flowers. D) why male orchid bees are smaller than female orchid bees. E) the importance of orchid and Brazil nut tree flowers for the production of orchid bee honey.

B) why Brazil nut trees do not set fruit in monoculture plantations

Which trait(s) is (are) shared by many modern gymnosperms and angiosperms? 1. pollen transported by wind 2. lignified xylem 3. microscopic gametophytes 4. sterile sporophylls, modified to attract pollinators 5. endosperm A) 1 only B) 1 and 3 C) 1, 2, and 3 D) 1, 3, and 5 E) 2, 4, and 5

C) 1, 2, and 3

@@@In which combination of locations would one who is searching for the gametophytes of flower of stone have the best chance of finding them? 1. moist soil 2. underground, nourished there by symbiotic fungi 3. south- or west-facing slopes 4. permanently shady places 5. far from any flower of stone sporophytes A) 1 only B) 1 and 2 C) 1, 2, and 4 D) 1, 2, and 5 E) 1, 3, 4, and 5

C) 1, 2, and 4

The microsporidian, Brachiola gambiae, parasitizes the mosquito, Anopheles gambiae. Adult female mosquitoes must take blood meals in order for their eggs to develop, and it is while they take blood that they transmit malarial parasites to humans. Male mosquitoes drink flower nectar. If humans are to safely and effectively use Brachiola gambiae as a biological control to reduce human deaths from malaria, then how many of the following statements should be true? 1. Brachiola should kill the mosquitoes before the malarial parasite they carry reaches maturity. 2. The microsporidian should not be harmful to other insects. 3. Microsporidians should infect mosquito larvae, rather than mosquito adults. 4. The subsequent decline in anopheline mosquitoes should not significantly disrupt human food resources or other food webs. 5. Brachiola must be harmful to male mosquitoes, but not to female mosquitoes. A) 2 and 5 B) 1, 2 and 4 C) 1, 2., and 4 D) 2, 3, 5 E) All of the statements are correct.

C) 1, 2., and 4

!!! Basing your inferences on information in the previous paragraph, to which clade(s) should cycliophorans belong? 1. Eumetazoa 2. Deuterostomia 3. Bilateria 4. Ecdysozoa 5. Lophotrochozoa A) 1 only B) 1 and 3 C) 1, 3, and 5 D) 2, 3, and 4 E) 2, 3, and 5

C) 1, 3, and 5

How many chromosomes should be in an embryo nucleus? A) 4 B) 8 C) 16 D) 24 E) 32

C) 16

@@@Upon closer inspection of the leaves of flower of stone, one can observe tiny, cone-like structures. Each cone-like structure emits spores of two different sizes. Further investigation also reveals that the roots of flower of stone branch only at the growing tip of the root, forming a Yshaped structure. Consequently, flower of stone should be expected to possess which other characteristics? 1. a gametophyte generation that is dominant 2. lignified vascular tissues 3. microphylls 4. filamentous rhizoids, but not true roots 5. spores that are diploid when mature A) 1 and 2 B) 1 and 5 C) 2 and 3 D) 2, 3, and 4 E) 3, 4, and 5

C) 2 and 3

Plasmogamy can directly result in which of the following? 1. cells with a single haploid nucleus 2. heterokaryotic cells 3. dikaryotic cells 4. cells with two diploid nuclei A) 1 or 2 B) 1 or 3 C) 2 or 3 D) 2 or 4 E) 3 or 4

C) 2 or 3

Fossil evidence indicates that the following events occurred in what sequence, from earliest to most recent? 1. Protostomes invade terrestrial environments. 2. Cambrian explosion occurs. 3. Deuterostomes invade terrestrial environments. 4. Vertebrates become top predators in the seas. A) 2 → 4 → 3 → 1 B) 2 → 1 → 4 → 3 C) 2 → 4 → 1 → 3 D) 2 → 3 → 1 → 4 E) 2 → 1 → 3 → 4

C) 2 → 4 → 1 → 3

Which of the following statements concerning animal taxonomy is (are) true? 1. Animals are more closely related to plants than to fungi. 2. All animal clades based on body plan have been found to be incorrect. 3. Kingdom Animalia is monophyletic. 4. Only animals reproduce by sexual means. 5. Animals are thought to have evolved from flagellated protists similar to modern choanoflagellates. A) 5 only B) 1 and 3 C) 3 and 5 D) 3, 4, and 5

C) 3 and 5

$$$ Which feature of cycads makes them similar to many angiosperms? 1. They have exposed ovules. 2. They have flagellated sperm. 3. They are pollinated by animals. A) 1 only B) 2 only C) 3 only D) 2 and 3 E) 1, 2, and 3

C) 3 only

Suppose that the cells of seed plants, like the skin cells of humans, produce a pigment upon increased exposure to UV radiation. Rank the following cells, from greatest to least, in terms of the likelihood of producing this pigment. 1. cells of sporangium 2. cells in the interior of a subterranean root 3. epidermal cells of sporophyte megaphylls 4. cells of a gametophyte A) 3, 4, 1, 2 B) 3, 4, 2, 1 C) 3, 1, 4, 2 D) 3, 2, 1, 4 E) 3, 1, 2, 4

C) 3, 1, 4, 2

###Animals that consume Brazil nuts are deriving nutrition mostly from tissue whose nuclei have how many chromosomes? A) 17 B) 34 C) 51 D) 68 E) There is not enough information to say.

C) 51

What is the most probable location of the oldest portion of this mycelium? (figure 26.2) A) A B) B C) C D) D

C) C

Which of the following animal groups is entirely aquatic? A) Mollusca B) Crustacea C) Echinodermata D) Nematoda E) Platyhelminthes

C) Echinodermata

Which of the following is the correct sequence of events in the origin of life? I. formation of protobionts II. synthesis of organic monomers III. synthesis of organic polymers IV. formation of DNA-based genetic systems

C) II, III, I, IV

*** Which phylogeny has been created by emphasizing genomic features of placozoans? A) I B) II C) III Answer:

C) III

Which tree depicts the closest relationship between zygomycetes and chytrids? A) I B) II C) III D) IV

C) III

The fruit of the mistletoe, a parasitic angiosperm, is a one-seeded berry. In members of the genus Viscum, the outside of the seed is viscous (sticky), which permits the seed to adhere to surfaces, such as the branches of host plants or the beaks of birds. What should be expected of the fruit if the viscosity of Viscum seeds is primarily an adaptation for dispersal rather than an adaptation for infecting host plant tissues? A) It should be drab in color. B) It should be colored so as to provide it with camouflage. C) It should be nutritious. D) It should secrete enzymes that can digest bark. E) It should contain chemicals that cause birds to fly to the ground and vomit.

C) It should be nutritious.

Which of the following best describes the physical relationship of the partners involved in lichens? A) Fungal cells are enclosed within algal cells. B) Lichen cells are enclosed within fungal cells. C) Photosynthetic cells are surrounded by fungal hyphae. D) The fungi grow on rocks and trees and are covered by algae. E) Algal cells and fungal cells mix together without any apparent structure.

C) Photosynthetic cells are surrounded by fungal hyphae.

When adult amphibian skin harbors populations of the bacterium, Janthinobacterium lividum (Jl), chytrid infection seems to be inhibited. Which of the following represents the best experimental design for conclusively determining whether this inhibition is real? A) Inoculate uninfected amphibians with Jl, and determine whether the amphibians continue to remain uninfected by chytrids. B) Inoculate infected amphibians with Jl, and determine whether the amphibians recover from infection by chytrids. C) Take infected amphibians and assign them to two populations. Leave one population alone; inoculate the other with Jl. Measure the rate at which infection proceeds in both populations. D) Take infected amphibians and assign them to two populations. Inoculate one population with a high dose of Jl; inoculate the other with a low dose of Jl. Measure the survival frequency in both populations.

C) Take infected amphibians and assign them to two populations. Leave one population alone; inoculate the other with Jl. Measure the rate at which infection proceeds in both populations.

All things being equal, which of these is the most parsimonious explanation for the change in the number of Hox genes from the last common ancestor of insects and vertebrates to ancestral vertebrates, as shown in Table 27.1? A) The occurrence of seven independent duplications of individual Hox genes. B) The occurrence of two distinct duplications of the entire seven-gene cluster, followed by the loss of one cluster. C) The occurrence of a single duplication of the entire seven-gene cluster.

C) The occurrence of a single duplication of the entire seven-gene cluster.

A researcher has developed two stains for use with seed plants. One stains sporophyte tissue blue; the other stains gametophyte tissue red. If the researcher exposes pollen grains to both stains, and then rinses away the excess stain, what should occur? A) The pollen grains will be pure red. B) The pollen grains will be pure blue. C) The pollen grains will have red interiors and blue exteriors. D) The pollen grains will have blue interiors and red exteriors. E) Insofar as the pollen grains are independent of the plant that produced them, they will not absorb either stain.

C) The pollen grains will have red interiors and blue exteriors

What is true of stamens, sepals, petals, carpels, and pinecone scales? A) They are female reproductive parts. B) None are capable of photosynthesis. C) They are modified leaves. D) They are found on flowers. E) They are found on angiosperms.

C) They are modified leaves.

What should be true of fossils of the earliest tetrapods? A) They should show evidence of internal fertilization. B) They should show evidence of having produced shelled eggs. C) They should indicate limited adaptation to life on land. D) They should be transitional forms with the fossils of chondrichthyans that lived at the same time. E) They should feature the earliest indications of the appearance of jaws.

C) They should indicate limited adaptation to life on land.

Which of the following statements is true of monocots? A) They are currently thought to be polyphyletic. B) The veins of their leaves form a netlike pattern. C) They, along with the eudicots, magnoliids, and basal angiosperms, are currently placed in the phylum Anthophyta. D) Each possesses multiple cotyledons. E) They are in the clade that includes most of our crops, except the cereal grains.

C) They, along with the eudicots, magnoliids, and basal angiosperms, are currently placed in the phylum Anthophyta.

Whatever its ultimate cause(s), the Cambrian explosion is a prime example of A) mass extinction. B) evolutionary stasis. C) adaptive radiation. D) a large meteor impact.

C) adaptive radiation.

Among the organisms listed here, which are thought to be the closest relatives of fungi? A) slime molds B) vascular plants C) animals D) brown algae E) mosses

C) animals

Which description does not apply equally well to both sexual and asexual spores? A) have haploid nuclei B) represent the dispersal stage C) are produced by meiosis D) upon germination, will subsequently undergo S phase and mitosis

C) are produced by meiosis

If nudibranch rhinophores are located at the anteriors of these sea slugs, then they contribute to the sea slugs' A) segmentation. B) lack of torsion. C) cephalization. D) identity as lophotrochozoans. E) ability to successfully carry out a sessile lifestyle.

C) cephalization.

According to our current knowledge of plant evolution, which group of organisms should feature mitosis most similar to that of land plants? A) unicellular green algae B) cyanobacteria C) charophytes D) red algae E) multicellular green algae

C) charophytes

Some researchers claim that sponge genomes have homeotic genes, but no Hox genes. If true, this finding would A) strengthen sponges' evolutionary ties to the Eumetazoa. B) mean that sponges must no longer be classified as animals. C) confirm the identity of sponges as "basal animals." D) mean that extinct sponges must have been the last common ancestor of animals and fungi. E) require sponges to be reclassified as choanoflagellates.

C) confirm the identity of sponges as "basal animals."

@@@ On the basis of the information in the previous paragraph, which of these should be able to be observed in Tp? A) the act of fertilization B) the process of gastrulation C) eggs D) All three of the responses above are correct. E) Two of the responses above are correct.

C) eggs

@@@Which of the following features is most important in order for true mosses and ferns to survive and reproduce in the desert? A) that the sporophytes occupy only permanently shady, north-facing habitats B) that the sporophytes hug the ground, growing no taller than a couple of inches C) either that their gametophytes grow close together, or that they be hermaphroditic D) that the sporophytes have highly lignified vascular tissues

C) either that their gametophytes grow close together, or that they be hermaphroditic

Which of the following terms refers to symbiotic relationships that involve fungi living between the cells in plant leaves? A) pathogens B) endosymbioses C) endophytes D) lichens E) mycorrhizae

C) endophytes

A trend first observed in the evolution of the earliest tetrapods was A) the appearance of jaws. B) the appearance of bony vertebrae. C) feet with digits. D) the mineralization of the endoskeleton. E) the amniotic egg.

C) feet with digits.

A botanist discovers a new species of plant in a tropical rain forest. After observing its anatomy and life cycle, he notes the following characteristics: flagellated sperm, xylem with tracheids, separate gametophyte and sporophyte generations with the sporophyte dominant, and no seeds. This plant is probably most closely related to A) mosses. B) charophytes. C) ferns. D) gymnosperms. E) flowering plants.

C) ferns.

Angiosperms are the most successful terrestrial plants. Which of the following features is unique to them and helps account for their success? A) wind pollination B) dominant gametophytes C) fruits enclosing seeds D) embryos enclosed within seed coats E) sperm cells without flagella

C) fruits enclosing seeds

When a mycelium infiltrates an unexploited source of dead organic matter, what are most likely to appear within the food source soon thereafter? A) fungal haustoria B) soredia C) fungal enzymes D) increased oxygen levels E) larger bacterial populations

C) fungal enzymes

When pathogenic fungi are found growing on the roots of grape vines, grape farmers sometimes respond by covering the ground around their vines with plastic sheeting and pumping a gaseous fungicide into the soil. The most important concern of grape farmers who engage in this practice should be that the A) fungicide might also kill the native yeasts residing on the surfaces of the grapes. B) lichens growing on the vines' branches are not harmed. C) fungicide might also kill mycorrhizae. D) sheeting is transparent so that photosynthesis can continue.

C) fungicide might also kill mycorrhizae.

Gymnosperms differ from both extinct and extant (living) ferns because they A) are woody. B) have macrophylls. C) have pollen. D) have sporophylls. E) have spores.

C) have pollen.

Which of the following is most important in making the typical seed more resistant to adverse conditions than the typical spore? A) a different type of sporopollenin B) an internal reservoir of liquid water C) integument(s) D) ability to be dispersed E) waxy cuticle

C) integument(s)

$$$ Which shark structure is most analogous to a swim bladder full of gas? A) its lateral line system B) its spiral valve C) its liver D) its dead-end nostrils E) its gills

C) its liver

Terry catches a ray-finned fish from the ocean and notices that attached to its flank is an equally long, snakelike organism. The attached organism has no external segmentation, no scales, a round mouth surrounded by a sucker, and two small eyes. Terry thinks it might be a marine leech, a hagfish, or a lamprey. Which feature excludes the organism from possibly being a leech? A) elongated shape B) lack of scales C) lack of external segmentation D) round mouth E) anterior sucker

C) lack of external segmentation

Which of the following has the least affiliation with all of the others? A) Glomeromycota B) mycorrhizae C) lichens D) arbuscules E) mutualistic fungi

C) lichens

Which clade does not include humans? A) synapsids B) lobe-fins C) lophotrochozoans D) tetrapods E) osteichthyans

C) lophotrochozoans

Which of these are amniotes? A) amphibians B) fishes C) mammals D) placental mammals only E) egg-laying mammals only

C) mammals

Which of the following is (are) unique to animals? A) cells that have mitochondria B) the structural carbohydrate, chitin C) nervous conduction and muscular movement D) heterotrophy E) flagellated gametes

C) nervous conduction and muscular movement

Which structure is common to both gymnosperms and angiosperms? A) stigma B) carpel C) ovule D) ovary E) anthers

C) ovule

In addition to seeds, which of the following characteristics is unique to the seed-producing plants? A) sporopollenin B) lignin present in cell walls C) pollen D) use of air currents as a dispersal agent E) megaphylls

C) pollen

The feeding mode of the extinct conodonts was A) herbivory. B) suspension feeding. C) predation. D) filter feeding. E) absorptive feeding.

C) predation.

If all fungi in an environment that perform decomposition were to suddenly die, then which group of organisms should benefit most, due to the fact that their fungal competitors have been removed? A) plants B) protists C) prokaryotes D) animals E) mutualistic fungi

C) prokaryotes

Corals are most closely related to which group? A) jellies B) freshwater hydras C) sea anemones D) sponges E) barnacles

C) sea anemones

%%% Had the teacher wanted to point out organisms that belong to the most successful animal phylum, the teacher should have chosen the A) bivalves and gastropods. B) sea anemones and hydra. C) shrimp and copepods. D) polychaete.

C) shrimp and copepods.

###The agouti is most directly involved with the Brazil nut tree's dispersal of A) male gametophytes. B) female gametophytes. C) sporophyte embryos. D) sporophyte megaspores. E) female gametes.

C) sporophyte embryos.

In seed plants, which of the following is part of a pollen grain and has a function most like that of the seed coat? A) sporophyll B) male gametophyte C) sporopollenin D) stigma E) sporangium

C) sporopollenin

The leaflike appendages of moss gametophytes may be one to two cell layers thick. Consequently, which of the following is least likely to be found associated with such appendages? A) cuticle B) rings of cellulose—synthesizing complexes C) stomata D) peroxisomes E) phenolics

C) stomata

Which statement about bacterial cell walls is false?

Cell walls prevent cells from dying in hypertonic conditions.

Foods can be preserved in many ways by slowing or preventing bacterial growth. Which of these methods should be least effective at inhibiting bacterial growth?

Closing previously opened containers: prevents more bacteria from entering, and excludes O2

$$$ When a shark stops swimming, it does which of the following? 1. sinks 2. quickly dies 3. oxygenates its blood less effectively A) 1 only B) 2 only C) 3 only D) 1 and 3 E) 1, 2, and 3

D) 1 and 3

Which of the following characterize at least some members of the phylum Cnidaria? 1. a gastrovascular cavity 2. a polyp stage 3. a medusa stage 4. cnidocytes 5. a pseudocoelom A) 1 B) 2 and 3 C) 3, 4, and 5 D) 1, 2, 3, and 4 E) All of the above are correct

D) 1, 2, 3, and 4

How many of the following statements about craniates is (are) correct? 1. Craniates are more highly cephalized than are noncraniate chordates. 2. Craniates' genomic evolution includes duplication of clusters of genes that code for transcription factors. 3. The craniate clade is synonymous with the vertebrate clade. 4. Pharyngeal slits that play a major role in gas exchange originated in craniates. 5. The two-chambered heart originated with the early craniates. A) only 1 is correct B) 1 and 5 are correct C) 2, 3, and 5 are correct D) 1, 2, 4, and 5 are correct E) all 5 options are correct

D) 1, 2, 4, and 5 are correct

!!!What is true of the feeding stage of cycliophorans? 1. It is chemoheterotrophic. 2. It is sessile. 3. It captures food in a manner similar to that of animals with lophophores. 4. It has radial symmetry. A) 1 and 2 B) 1 and 3 C) 2 and 4 D) 1, 2, and 3 E) 2, 3, and 4

D) 1, 2, and 3

Sexual reproduction has not been observed in Bd. A Bd sporangium initially contains a single, haploid cell. Which of the following processes must be involved in generating the multiple zoospores eventually produced by each sporangium? 1. S phase 2. cytokinesis 3. mitosis 4. meiosis A) 1 and 2 B) 1 and 3 C) 2 and 3 D) 1, 2, and 3 E) 1, 2, and 4

D) 1, 2, and 3

Which of the following were probably factors that permitted early plants to successfully colonize land? 1. the relative number of potential predators (herbivores) 2. the relative number of competitors 3. the relative availability of symbiotic partners 4. air's relative lack of support, compared to water's support A) 1 and 2 B) 2 and 3 C) 3 and 4 D) 1, 2, and 3 E) 1, 2, and 4

D) 1, 2, and 3

If intelligent extraterrestrials visited Earth 475 million years ago, and then again 300 million years ago (at the close of the Carboniferous period), what trends would they have noticed in Earth's terrestrial vegetation over this period? 1. a trend from dominant gametophytes to dominant sporophytes 2. a trend from sporangia borne on modified leaves (sporophylls) to sporangia borne on stalks (seta) 3. a trend from no true leaves, to microphylls, to megaphylls 4. a trend from soil-surface-hugging plants to "overtopping" plants 5. a trend toward increased lignification of conducting systems A) 1 and 3 B) 3, 4, and 5 C) 1, 2, 4, and 5 D) 1, 3, 4, and 5 E) 2, 3, 4, and 5

D) 1, 3, 4, and 5

$$$ Which feature of cycads distinguishes them from most other gymnosperms? 1. They have exposed ovules. 2. They have flagellated sperm. 3. They are pollinated by animals. A) 1 only B) 2 only C) 3 only D) 2 and 3 E) 1, 2, and 3

D) 2 and 3

@@@Which of the following characteristics is (are) possessed in common by true mosses, ferns, and spike mosses, and therefore becomes useless at helping to determine to which of these groups flower of stone belongs? 1. a sporophyte generation that is dominant 2. true leaves and roots 3. flagellated sperm 4. strobili 5. alternation of generations A) 5 only B) 1 and 5 C) 2 and 3 D) 3 and 5 E) 2, 4, and 5

D) 3 and 5

What adaptations should one expect of the seed coats of angiosperm species whose seeds are dispersed by frugivorous (fruit-eating) animals, as opposed to angiosperm species whose seeds are dispersed by other means? 1. The exterior of the seed coat should have barbs or hooks. 2. The seed coat should contain secondary compounds that irritate the lining of the animal's mouth. 3. The seed coat should be able to withstand low pH's. 4. The seed coat, upon its complete digestion, should provide vitamins or nutrients to animals. 5. The seed coat should be resistant to the animals' digestive enzymes. A) 4 only B) 1 and 2 C) 2 and 3 D) 3 and 5 E) 3, 4, and 5

D) 3 and 5

The Hox genes came to regulate each of the following in what sequence, from earliest to most recent? 1. identity and position of paired appendages in protostome embryos 2. anterior-posterior orientation of segments in protostome embryos 3. positioning of tentacles in cnidarians 4. anterior-posterior orientation in vertebrate embryos A) 4 → 1 → 3 → 2 B) 4 → 2 → 3 → 1 C) 4 → 2 → 1 → 3 D) 3 → 2 → 1 → 4 E) 3 → 4 → 1 → 2

D) 3 → 2 → 1 → 4

Assuming that they all belong to the same plant, arrange the following structures from largest to smallest. 1. antheridia 2. gametes 3. gametophytes 4. gametangia A) 1, 4, 3, 2 B) 3, 1, 2, 4 C) 3, 4, 2, 1 D) 3, 4, 1, 2 E) 4, 3, 1, 2

D) 3, 4, 1, 2

Arrange these taxonomic terms from most inclusive (most general) to least inclusive (most specific). 1. lobe-fins 2. amphibians 3. gnathostomes 4. osteichthyans 5. tetrapods A) 4, 3, 1, 5, 2 B) 4, 3, 2, 5, 1 C) 4, 2, 3, 5, 1 D) 3, 4, 1, 5, 2 E) 3, 4, 5, 1, 2

D) 3, 4, 1, 5, 2

The vegetative (nutritionally active) bodies of most fungi are A) composed of hyphae. B) referred to as a mycelium. C) usually underground. D) All of these responses are correct.

D) All of these responses are correct.

Which of the following is a true statement about angiosperm carpels? A) Carpels are features of the gametophyte generation. B) Carpels consist of anther and stamen. C) Carpels are structures that directly produce male gametes. D) Carpels surround and nourish the female gametophyte. E) Carpels consist of highly modified microsporangia.

D) Carpels surround and nourish the female gametophyte.

Which of the following statements is true of the pine life cycle? A) Cones are homologous to the capsules of moss plants. B) The pine tree is a gametophyte. C) Male and female gametophytes are in close proximity during gamete synthesis. D) Conifer pollen grains contain male gametophytes. E) Double fertilization is a relatively common phenomenon.

D) Conifer pollen grains contain male gametophytes.

Assume that all four locations are 0.5 m above the surface. On a breezy day with prevailing winds blowing from left to right, where should one expect to find the highest concentration of free basidiospores in an air sample? (figure 26.2) A) A B) B C) C D) D

D) D

$$$ On the Pacific island of Guam, large herbivorous bats called "flying foxes" commonly feed on cycad seeds, a potent source of neurotoxins. The flying foxes do not visit male cones. Consequently, what should be true? A) The flying foxes are attracted to cycad fruit, and eat the enclosed seeds only by accident. B) Flying foxes are highly susceptible to the effects of the neurotoxins. C) The flying foxes assist the beetles as important pollinating agents of the cycads. D) Flying foxes can be dispersal agents of cycad seeds if the seeds sometimes get swallowed whole (in other words, without getting chewed).

D) Flying foxes can be dispersal agents of cycad seeds if the seeds sometimes get swallowed whole (in other words, without getting chewed).

A team of researchers has developed a poison that has proven effective against lamprey larvae in freshwater cultures. The poison is ingested and causes paralysis by detaching segmental muscles from the skeletal elements. The team wants to test the poison's effectiveness in streams feeding Lake Michigan, but one critic worries about potential effects on lancelets, which are similar to lampreys in many ways. Why is this concern misplaced? A) A chemical poisonous to lampreys could not also be toxic to organisms as ancestral as lancelets. B) Lamprey larvae and lancelets have very different feeding mechanisms. C) Lancelets do not have segmental muscles. D) Lancelets live only in saltwater environments. E) Lancelets and lamprey larvae eat different kinds of food.

D) Lancelets live only in saltwater environments.

Considering that the mature sporophytes of true mosses get their nutrition from the gametophytes on which they grow, and considering these generations as individual plants, what is true of the relationship between true moss sporophytes and gametophytes? A) Sporophytes are endosymbionts of gametophytes. B) Sporophytes are mutualists of gametophytes. C) Sporophytes are commensalists of gametophytes. D) Sporophytes are parasites of gametophytes.

D) Sporophytes are parasites of gametophytes.

Which of the following statements would be least acceptable to most zoologists? A) The extant lancelets are contemporaries, not ancestors, of vertebrates. B) The first fossils resembling lancelets appeared in the fossil record around 530 million years ago. C) Recent work in molecular systematics supports the hypothesis that lancelets are the basal clade of chordates. D) The extant lancelets are the immediate ancestors of the fishes. E) Lancelets display the same method of swimming as do fishes.

D) The extant lancelets are the immediate ancestors of the fishes.

Which of the following is a characteristic of hyphate fungi (fungi featuring hyphae)? A) They acquire their nutrients by phagocytosis. B) Their body plan is a unicellular sphere. C) Their cell walls consist mainly of cellulose microfibrils. D) They are adapted for rapid directional growth to new food sources. E) They reproduce asexually by a process known as budding.

D) They are adapted for rapid directional growth to new food sources

Which of the following is true of members of the phylum Cnidaria? A) They are not capable of locomotion because they lack true muscle tissue. B) They are primarily filter feeders. C) They have either, or both, of two body forms: mobile polyps and sessile medusae. D) They may use a gastrovascular cavity as a hydrostatic skeleton. E) They are the simplest organisms with a complete alimentary canal (two openings).

D) They may use a gastrovascular cavity as a hydrostatic skeleton.

Which characteristic(s) is (are) shared by both cnidarians and flatworms? A) dorsoventrally flattened bodies B) true muscle C) radial symmetry D) a digestive system with a single opening E) two of these

D) a digestive system with a single opening

If infection primarily involves the outermost layers of adult amphibian skin, and if the chytrids use the skin as their sole source of nutrition, then which term best applies to the chytrids? A) anaerobic chemoautotroph B) aerobic chemoautotroph C) anaerobic chemoheterotroph D) aerobic chemoheterotroph

D) aerobic chemoheterotroph

A sponge's structural materials (spicules, spongin) are manufactured by the A) pore cells. B) epidermal cells. C) choanocytes. D) amoebocytes.

D) amoebocytes.

The decline of cyanobacteria in the early Cambrian oceans was most likely related to A) the movement of animals onto land. B) the evolution of land plants. C) an increase in the number of predatory fish. D) an increase in filter-feeding organisms. E) a decline in the number of marine decomposers.

D) an increase in filter-feeding organisms.

Which of the following are the only extant animals that descended directly from dinosaurs? A) lizards B) crocodiles C) snakes D) birds E) tuataras

D) birds

All of the following cellular structures are functionally important in cells of the gametophytes of both angiosperms and gymnosperms except A) haploid nuclei. B) mitochondria. C) cell walls. D) chloroplasts. E) peroxisomes.

D) chloroplasts.

Fossil fungi date back to the origin and early evolution of plants. What combination of environmental and morphological change is similar in the evolution of both fungi and plants? A) presence of "coal forests" and change in mode of nutrition B) periods of drought and presence of filamentous body shape C) predominance in swamps and presence of cellulose in cell walls D) colonization of land and loss of flagellated cells E) continental drift and mode of spore dispersal

D) colonization of land and loss of flagellated cells

Immediately after karyogamy occurs, which term applies? A) plasmogamy B) heterokaryotic C) dikaryotic D) diploid

D) diploid

!!! Cycliophorans have two types of larvae. One type of larva is produced when the digestive system of a female is impregnated by a male. The digestive system then collapses and develops into a larva, which swims away in search of a new host after the surrounding female dies. Which is the embryonic tissue that is apparently most important in forming this type of larva? A) mesohyl B) mesoderm C) ectoderm D) endoderm E) mesoglea

D) endoderm

Which of the following sex and generation combinations most directly produces the integument of a pine seed? A) male gametophyte B) female gametophyte C) male sporophyte D) female sporophyte

D) female sporophyte

Which of the following sex and generation combinations most directly produces the megasporangium of pine ovules? A) male gametophyte B) female gametophyte C) male sporophyte D) female sporophyte

D) female sporophyte

Which of the following sex and generation combinations most directly produces the fruit? A) male gametophyte B) female gametophyte C) male sporophyte D) female sporophyte

D) female sporophyte

Sponges are most accurately described as A) marine predators. B) chemoautrophs. C) freshwater scavengers. D) filter feeders. E) aquatic predators.

D) filter feeders.

Which of the following genetic processes may be most helpful in accounting for the Cambrian explosion? A) binary fission B) mitosis C) random segregation D) gene duplication E) chromosomal condensation

D) gene duplication

Which of these paired fungal structures are structurally and functionally most alike? A) conidia and basidiocarps B) sporangia and hyphae C) soredia and gills D) haustoria and arbuscules E) zoospores and mycelia

D) haustoria and arbuscules

A new, sixth global mass-extinction event appears to be occurring on Earth today. The most likely explanation for the dramatic loss of species is A) global warming. B) natural fluctuations in global temperatures and precipitation. C) the evolution of new types of viruses. D) human activity that alters natural environments. E) decreases in atmospheric levels of oxygen.

D) human activity that alters natural environments.

%%% One day, little Tommy (a student in an undersupervised class of 40 fifth graders) got the urge to pet Nemo (the clownfish), who was swimming among the waving petals of a pretty underwater "flower" that had a big hole in the midst of the petals. Tommy giggled upon finding that these petals felt sticky. A few hours later, Tommy was in the nurse's office with nausea and cramps. Microscopic examination of his fingers would probably have revealed the presence of A) teeth marks. B) spines. C) spicules. D) nematocysts. E) a radula.

D) nematocysts.

Which of the following flower parts develops into the pulp of a fleshy fruit? A) stigma B) style C) ovule D) ovary E) micropyle

D) ovary

What do all craniates have that earlier chordates did not have? A) brain B) vertebrae C) cartilaginous pipe surrounding notochord D) partial or complete skull E) bone

D) partial or complete skull

Which of the following can be found in gymnosperms? A) nonfertile flower parts B) triploid endosperm C) fruits D) pollen E) carpels

D) pollen

What do animals as diverse as corals and monkeys have in common? A) body cavity between body wall and digestive system B) number of embryonic tissue layers C) type of body symmetry D) presence of Hox genes E) degree of cephalization

D) presence of Hox genes

The swim bladder of ray-finned fishes A) was probably modified from simple lungs of chondrichthyans. B) developed into lungs in saltwater fishes. C) first appeared in sharks. D) provides for regulation of buoyancy. E) Two of the options listed are correct.

D) provides for regulation of buoyancy.

The structural integrity of bacteria is to peptidoglycan as the structural integrity of plant spores is to A) lignin. B) cellulose. C) secondary compounds. D) sporopollenin.

D) sporopollenin.

Which of the following characteristics helped seedless plants better adapt to life on land? A) a dominant gametophyte B) photosystem II C) a chitinous cuticle D) stomata E) an unbranched sporophyte

D) stomata

Which of the following was probably the least important factor in bringing about the Cambrian explosion? A) the emergence of predator-prey relationships among animals B) the accumulation of diverse adaptations, such as shells and different modes of locomotion C) the origin of Hox genes and other genetic changes affecting the regulation of developmental genes D) the movement of animals onto land E) the accumulation of sufficient atmospheric oxygen to support the more active metabolism of mobile animals

D) the movement of animals onto land

The most ancient branch point in animal phylogeny is that between having A) radial or bilateral symmetry. B) a well-defined head or no head. C) diploblastic or triploblastic embryos. D) true tissues or no tissues. E) a body cavity or no body cavity.

D) true tissues or no tissues.

In both lichens and mycorrhizae, what does the fungal partner provide to its photosynthetic partner? A) carbohydrates B) fixed nitrogen C) antibiotics D) water and minerals E) protection from harmful UV

D) water and minerals

Among the invertebrate phyla, phylum Arthropoda is unique in possessing members that have A) a cuticle. B) a ventral nerve cord. C) open circulation. D) wings. E) segmented bodies.

D) wings.

The best evidence for not classifying the slime molds as fungi comes from slime molds' A) DNA sequences. B) nutritional modes. C) choice of habitats. D) physical appearance. E) reproductive methods.

DNA sequences

!!! Using similarities in embryonic development, body symmetry, and other anatomical features to assign an organism to a clade involves 1. cladistics based on body plan. 2. molecular-based phylogeny. 3. morphology-based phylogeny. A) 1 only B) 2 only C) 3 only D) 1 and 2 E) 1 and 3

E) 1 and 3

The chytrid sporangia reside within the amphibian epidermal cells. Consequently, which term(s) apply to Bd? 1. ectosymbionts 2. parasites 3. commensals 4. pathogens 5. endosymbionts A) 1 and 2 B) 1 and 3 C) 2 and 4 D) 2, 3, and 5 E) 2, 4, and 5

E) 2, 4, and 5

Arrange the following in the correct sequence, from earliest to most recent, in which these plant traits originated. 1. sporophyte dominance, gametophyte independence 2. sporophyte dominance, gametophyte dependence 3. gametophyte dominance, sporophyte dependence A) 1 → 2 → 3 B) 2 → 3 → 1 C) 2 → 1 → 3 D) 3 → 2 → 1 E) 3 → 1 → 2

E) 3 → 1 → 2

$$$ Rank the following fish, from most to least, in terms of the amount of energy each must use to maintain its position (depth) in the water column over the long term. 1. physoclistus fish 2. physostomus fish 3. chondrichthyan fish A) 1, 2, 3 B) 2, 3, 1 C) 2, 1, 3 D) 3, 1, 2 E) 3, 2, 1

E) 3, 2, 1

Assuming that they all belong to the same plant, arrange the following structures from largest to smallest (or from most inclusive to least inclusive). 1. spores 2. sporophylls 3. sporophytes 4. sporangia A) 2, 4, 3, 1 B) 2, 3, 4, 1 C) 3, 1, 4, 2 D) 3, 4, 2, 1 E) 3, 2, 4, 1

E) 3, 2, 4, 1

Arrange the following structures from largest to smallest, assuming that they belong to two generations of the same angiosperm. 1. ovary 2. ovule 3. egg 4. carpel 5. embryo sac A) 4, 2, 1, 5, 3 B) 4, 5, 2, 1, 3 C) 5, 4, 3, 1, 2 D) 5, 1, 4, 2, 3 E) 4, 1, 2, 5, 3

E) 4, 1, 2, 5, 3

Which of these would a paleontologist be most likely to do in order to determine whether a fossil represents a reptile or a mammal? A) Look for the presence of milk-producing glands. B) Look for the mammalian characteristics of a four-chambered heart and a diaphragm. C) Because mammals are eutherians, look for evidence of a placenta. D) Use molecular analysis to look for the protein keratin. E) Examine the teeth.

E) Examine the teeth.

Each of the eight ascospores present at the end of mitosis has the same chromosome number and DNA content (ng) as each of the four cells at the end of meiosis. What must have occurred in each spore between the round of meiosis and the round of mitosis? A) double fertilization B) crossing over C) nondisjunction D) autopolyploidy E) S phase

E) S phase

The fact that infection by Bd causes lethargy in many infected amphibians can have what effect on efforts to accurately census the numbers of dead or dying amphibians at a particular time, in a particular habitat? A) It can cause underestimation, due to infected amphibians preferring to seek out refuges relative to uninfected amphibians. B) It can cause underestimation, due to increased predation on, and removal of, infected amphibians relative to uninfected amphibians. C) It can cause overestimation, because infected frogs should be more readily observable to human census-takers than should uninfected amphibians. D) All three of the above statements are plausible. E) Two of the above statements are plausible.

E) Two of the above statements are plausible.

Which of the following characteristics of plants is absent in their closest relatives, the charophyte algae? A) chlorophyll b B) cellulose in cell walls C) multicellularity D) sexual reproduction E) alternation of generations

E) alternation of generations

The adaptive advantage associated with the filamentous nature of fungal mycelia is primarily related to A) the ability to form haustoria and parasitize other organisms. B) avoiding sexual reproduction until the environment changes. C) the potential to inhabit almost all terrestrial habitats. D) the increased probability of contact between different mating types. E) an extensive surface area well suited for invasive growth and absorptive nutrition.

E) an extensive surface area well suited for invasive growth and absorptive nutrition.

Arthropods invaded land about 100 million years before vertebrates did so. This most clearly implies that A) arthropods evolved before vertebrates did. B) extant terrestrial arthropods are better adapted to terrestrial life than are extant terrestrial vertebrates. C) ancestral arthropods must have been poorly adapted to aquatic life, and thus experienced a selective pressure to invade land. D) vertebrates evolved from arthropods. E) arthropods have had more time to coevolve with land plants than have vertebrates.

E) arthropods have had more time to coevolve with land plants than have vertebrates.

Fossil steroid and molecular clock evidence suggests that animals originated A) between 770 and 710 million years ago. B) more than 100 million years before the oldest known fossils of large animals. C) during the Cambrian explosion. D) after sponges diverged from other metazoans. E) both A and B

E) both A and B

To which of the following are the scales of chondrichthyans most closely related in a structural sense? A) osteichthyan scales B) reptilian scales C) mammalian scales D) bird scales E) chondrichthyan teeth

E) chondrichthyan teeth

On a field trip, a student in a marine biology class collects an organism that has differentiated organs, cell walls of cellulose, and chloroplasts with chlorophyll a. Based on this description, the organism could be a brown alga, a red alga, a green alga, a charophyte recently washed into the ocean from a freshwater or brackish water source, or a land plant washed into the ocean. The presence of which of the following features would definitively identify this organism as a land plant? A) alternation of generations B) sporopollenin C) rings of cellulose-synthesizing complexes D) flagellated sperm E) embryos

E) embryos

In terms of alternation of generations, the internal parts of the pollen grains of seedproducing plants are most similar to a A) moss sporophyte. B) moss gametophyte bearing both male and female gametangia. C) fern sporophyte. D) hermaphroditic fern gametophyte. E) fern gametophyte bearing only antheridia.

E) fern gametophyte bearing only antheridia.

The last common ancestor of all animals was probably a A) unicellular chytrid. B) unicellular yeast. C) multicellular algae. D) multicellular fungus. E) flagellated protist.

E) flagellated protist.

Lampreys differ from hagfishes in A) lacking jaws. B) having a cranium. C) having pharyngeal clefts that develop into pharyngeal slits. D) having a notochord throughout life. E) having a notochord that is surrounded by a tube of cartilage.

E) having a notochord that is surrounded by a tube of cartilage.

What is the greatest threat to plant diversity? A) insects B) grazing and browsing by animals C) pathogenic fungi D) competition with other plants E) human population growth

E) human population growth

Which statement is most consistent with the hypothesis that the Cambrian explosion was caused by the rise of predator—prey relationships? A) increased incidence of worm burrows in the fossil record B) increased incidence of larger animals in the fossil record C) increased incidence of organic material in the fossil record D) increased incidence of fern galls in the fossil record E) increased incidence of hard parts in the fossil record

E) increased incidence of hard parts in the fossil record

Many mammals have skins and mucous membranes that are sensitive to phenolic secretions of plants like poison oak (Rhus). These secondary compounds are primarily adaptations that A) prevent desiccation. B) favor pollination. C) foster seed dispersal. D) decrease competition. E) inhibit herbivory.

E) inhibit herbivory.

The hydrolytic digestion of which of the following should produce monomers that are aminated (i.e., have an amine group attached) molecules of β-glucose? A) insect exoskeleton B) plant cell walls C) fungal cell walls D) insect exoskeleton and fungal cell walls E) insect exoskeleton and plant cell walls

E) insect exoskeleton and plant cell walls

&&& The shell of an animal egg is what type of angiosperm analog? A) endosperm B) pollen tube and sperm nuclei C) carpels D) fruit E) integuments

E) integuments

Reptilian embryos are protected from desiccation by a leathery shell. Similarly, which pair of structures protects seed plants' embryos and male gametophytes, respectively, from desiccation? A) ovules - waxy cuticle B) ovaries - filaments C) fruits - stamens D) pollen grains - waxy cuticle E) integuments - sporopollenin

E) integuments - sporopollenin

Which process occurs in fungi and has the opposite effect on a cell's chromosome number than does meiosis I? A) mitosis B) plasmogamy C) crossing over D) binary fission E) karyogamy

E) karyogamy

***This student has probably found a(n) A) immature pine tree. B) bryophyte sporophyte. C) fern sporophyte. D) horsetail gametophyte. E) lycophyte sporophyte.

E) lycophyte sporophyte.

$$$ The presence of a swim bladder allows the typical ray-finned fish to stop swimming and still A) effectively circulate its blood. B) be highly maneuverable. C) use its lateral line system. D) use its swim bladder as a respiratory organ. E) not sink.

E) not sink.

Which of the following factors, when used to label the horizontal axis of the graph in Figure 27.1, would account most directly for the shape of the plot? A) spongin concentration (gm/unit volume) B) rate of cribrostatin synthesis (molecules/unit time) C) number of pores per sponge D) number of spicules per sponge E) number of choanocytes per sponge

E) number of choanocytes per sponge

### Entrepreneurs attempted, but failed, to harvest nuts from plantations grown in Southeast Asia. Attempts to grow Brazil nut trees in South American plantations also failed. In both cases, the trees grew vigorously, produced healthy flowers in profusion, but set no fruit. Consequently, what is the likely source of the problem? A) poor sporophyte viability B) poor sporophyte fertility C) failure to produce fertile ovules D) failure to produce pollen E) pollination failure

E) pollination failure

The seed coat's most important function is to provide A) a nonstressful environment for the megasporangium. B) the means for dispersal. C) dormancy. D) a nutrient supply for the embryo. E) protection for the embryo.

E) protection for the embryo.

A botanist discovers a new species of land plant with a dominant sporophyte, chlorophylls a and b, and cell walls made of cellulose. In assigning this plant to a phylum, which of the following, if present, would be least useful? A) endosperm B) seeds C) sperm that lack flagella D) flowers E) spores

E) spores

Which of the following belong to the lobe-fin clade? A) chondrichthyans B) ray-finned fishes C) lampreys D) hagfishes E) tetrapods

E) tetrapods

You are hiking in a forest and happen upon a plant featuring a central stemlike structure from which sprout many, tiny, leaflike structures. Which of the following would be the most certain means of distinguishing whether it was a true moss or a club moss? A) its color B) its height C) if seeds are present D) if conducting tissues are present E) the appearance of its spore-producing structures

E) the appearance of its spore-producing structures

There is evidence that ray-finned fishes evolved A) in response to a crisis that wiped out the chondrichthyans. B) directly from lampreys and hagfish. C) early in the Cambrian period. D) directly from lancelets. E) the swim bladder from a lung.

E) the swim bladder from a lung.

Which statement about the domain Archaea is true? A) Genetic prospecting has recently revealed the existence of many previously unknown archaean species. B) No archaeans can reduce CO2 to methane. C) The genomes of archaeans are unique, containing no genes that originated within bacteria. D) No archaeans can inhabit solutions that are nearly 30% salt. E) No archaeans are adapted to waters with temperatures above the boiling point.

Genetic prospecting has recently revealed the existence of many previously unknown archaean species.

Several scientific laboratories across the globe are involved in research concerning the origin of life on Earth. Which of these questions is currently the most problematic and would have the greatest impact on our understanding if we were able to answer it?

How did RNA sequences come to carry the code for amino acid sequences?

Diatoms are mostly asexual members of the phytoplankton. Diatoms lack any organelles that might have the 9 + 2 pattern. They obtain their nutrition from functional chloroplasts, and each diatom is encased within two porous, glasslike valves. Which question would be most important for one interested in the day-to-day survival of individual diatoms? A) How does carbon dioxide get into these protists with their glasslike valves? B) How do diatoms get transported from one location on the water's surface layers to another location on the surface? C) How do diatoms with their glasslike valves keep from sinking into poorly lit waters? D) How do diatoms with their glasslike valves avoid being shattered by the action of waves? E) How do diatom sperm cells locate diatom egg cells?

How do diatoms with their glasslike valves keep from sinking into poorly lit waters?

If it were possible to conduct sophisticated microscopic and chemical analyses of microfossils found in 3.2-billion-year-old stromatolites, then one should be surprised to observe evidence of which of the following within such microfossils? I. double-stranded DNA II. a nuclear envelope III. a nucleoid IV. a nucleolus V. ribosomes

II and IV

Chloramphenicol is an antibiotic that targets prokaryotic (70S) ribosomes, but not eukaryotic (80S) ribosomes. Which of these questions stems from this observation, plus an understanding of eukaryotic origins? A) Can chloramphenicol also be used to control human diseases that are caused by archaeans? B) Can chloramphenicol pass through the capsules possessed by many cyanobacteria? C) If chloramphenicol inhibits prokaryotic ribosomes, should it not also inhibit mitochondrial ribosomes? D) Why aren't prokaryotic ribosomes identical to eukaryotic ribosomes? E) How is translation affected in ribosomes that are targeted by chloramphenicol?

If chloramphenicol inhibits prokaryotic ribosomes, should it not also inhibit mitochondrial ribosomes?

Why is the filamentous morphology of the water molds considered a case of convergent evolution with fungi? A) Water molds evolved from filamentous fungi. B) Body shape reflects ancestor—descendant relationships among organisms. C) In both cases, filamentous shape is an adaptation for the absorptive nutritional mode of a decomposer. D) Filamentous body shape is evolutionarily ancestral for all eukaryotes. E) Both the first and second responses above are correct.

In both cases, filamentous shape is an adaptation for the absorptive nutritional mode of a decomposer.

An individual mixotroph loses its plastids, yet continues to survive. Which of the following most likely accounts for its continued survival? A) It relies on photosystems that float freely in its cytosol. B) It must have gained extra mitochondria when it lost its plastids. C) It engulfs organic material by phagocytosis or by absorption. D) It has an endospore. E) It is protected by a case made of silica.

It engulfs organic material by phagocytosis or by absorption

If the Archaeplastidae are eventually designated a kingdom, and if land plants are excluded from this kingdom, then what will be true of this new kingdom? A) It will be monophyletic. B) It will more accurately depict evolutionary relationships than does the current taxonomy. C) It will be paraphyletic. D) It will be a true clade. E) It will be polyphyletic.

It will be paraphyletic

Similar to most amoebozoans, the forams and the radiolarians also have pseudopods, as do some of the white blood cells of animals (monocytes). If one were to construct a taxon that included all organisms that have cells with pseudopods, what would be true of such a taxon? A) It would be polyphyletic. B) It would be paraphyletic. C) It would be monophyletic. D) It would include all eukaryotes.

It would be polyphyletic

Regarding prokaryotic genetics, which statement is correct? A) Crossing over during prophase I introduces some genetic variation. B) Prokaryotes feature the union of haploid gametes, as do eukaryotes. C) Prokaryotes exchange some of their genes by conjugation, the union of haploid gametes, and transduction. D) Mutation is a primary source of variation in prokaryote populations. E) Prokaryotes skip sexual life cycles because their life cycle is too short.

Mutation is a primary source of variation in prokaryote populations

Which of the following statements is not true? A) Archaea and bacteria have different membrane lipids. B) Both archaea and bacteria generally lack membrane enclosed organelles. C) The cell walls of archaea lack peptidoglycan. D) Only bacteria have histones associated with DNA. E) Only some archaea use CO2 to oxidize H2, releasing methane.

Only bacteria have histones associated with DNA

How were conditions on the early Earth of more than 3 billion years ago different from those on today's Earth?

Only early Earth was intensely bombarded by large space debris.

Which statement about the genomes of prokaryotes is correct? A) Prokaryotic genomes are diploid throughout most of the cell cycle. B) Prokaryotic chromosomes are sometimes called plasmids. C) Prokaryotic cells have multiple chromosomes, "packed" with a relatively large amount of protein. D) The prokaryotic chromosome is not contained within a nucleus but, rather, is found at the nucleolus. E) Prokaryotic genomes are composed of circular DNA.

Prokaryotic genomes are composed of circular DNA.

Prokaryotic ribosomes differ from those present in eukaryotic cytosol. Because of this, which of the following is correct?

Some antibiotics can block protein synthesis in bacteria without effects in the eukaryotic host.

Living diatoms contain brownish plastids. If global warming causes blooms of diatoms in the surface waters of Earth's oceans, how might this be harmful to the animals that build coral reefs? A) The coral animals, which capture planktonic organisms, may be outcompeted by the diatoms. B) The coral animals' endosymbiotic dinoflagellates may get "shaded out" by the diatoms. C) The coral animals may die from overeating the plentiful diatoms, with their cases of silica. D) The diatoms' photosynthetic output may over-oxygenate the water.

The coral animals' endosymbiotic dinoflagellates may get "shaded out" by the diatoms.

Which of the following statements provides the strongest evidence that prokaryotes evolved before eukaryotes?

The oldest fossilized cells resemble prokaryotes

If all prokaryotes on Earth suddenly vanished, which of the following would be the most likely and most direct result? A) The number of organisms on Earth would decrease by 10—20%. B) Human populations would thrive in the absence of disease. C) Bacteriophage numbers would dramatically increase. D) The recycling of nutrients would be greatly reduced, at least initially. E) There would be no more pathogens on Earth.

The recycling of nutrients would be greatly reduced, at least initially.

What is true of the amino acids that might have been delivered to Earth within carbonaceous chondrites?

There were more kinds of amino acids on the chondrites than are found in living organisms today.

Mycoplasmas are bacteria that lack cell walls. On the basis of this structural feature, which statement concerning mycoplasmas should be true? A) They are gram-negative. B) They are subject to lysis in hypotonic conditions. C) They lack a cell membrane as well. D) They should contain less cellulose than do bacteria that possess cell walls. E) They possess typical prokaryotic flagella.

They are subject to lysis in hypotonic conditions.

Which of these statements about prokaryotes is correct? A) Bacterial cells conjugate to mutually exchange genetic material. B) Their genetic material is confined within vesicles known as plasmids. C) They divide by binary fission, without mitosis or meiosis. D) The persistence of bacteria throughout evolutionary time is due to their genetic homogeneity (in other words, sameness). E) Genetic variation in bacteria is not known to occur, because of their asexual mode of reproduction.

They divide by binary fission, without mitosis or meiosis.

Which of the following statements about dinoflagellates is true? A) They possess two flagella. B) All known varieties are autotrophic. C) Their walls are usually composed of silica plates. D) Many types lack mitochondria. E) Their dead cells accumulate on the seafloor, and are mined to serve as a filtering material.

They possess two flagella

The typical prokaryotic flagellum features

a complex "motor" embedded in the cell wall and plasma membrane.

Which of the following is a defining characteristic that all protobionts had in common?

a surrounding membrane or membrane-like structure

Biologists think that endosymbiosis gave rise to mitochondria before plastids partly because A) the products of photosynthesis could not be metabolized without mitochondrial enzymes. B) all eukaryotes have mitochondria (or their remnants), whereas many eukaryotes do not have plastids. C) mitochondrial DNA is less similar to prokaryotic DNA than is plastid DNA. D) without mitochondrial CO2 production, photosynthesis could not occur. E) mitochondrial proteins are synthesized on cytosolic ribosomes, whereas plastids utilize their own ribosomes.

all eukaryotes have mitochondria (or their remnants), whereas many eukaryotes do not have plastids

Which of the following are actual mutualistic partnerships that involve a protist and a host organism? A) cellulose-digesting gut protists—wood-eating termites B) dinoflagellates—reef-building coral animals C) Trichomonas—humans D) algae—certain foraminiferans E) all except that involving humans

all except that involving humans

Green algae differ from land plants in that many green algae: A) are heterotrophs. B) are unicellular. C) have plastids. D) have alternation of generations. E) have cell walls containing cellulose.

are unicellular

The first genes on Earth were probably

auto-catalytic RNA molecules.

Which of the following is least associated with the others? A) horizontal gene transfer B) genetic recombination C) conjugation D) transformation E) binary fission

binary fission

In general, what is the primary ecological role of prokaryotes? A) parasitizing eukaryotes, thus causing diseases B) breaking down organic matter C) metabolizing materials in extreme environments D) adding methane to the atmosphere E) serving as primary producers in terrestrial environments

breaking down organic matter


Kaugnay na mga set ng pag-aaral

Science Final Chemistry Questions

View Set

PSYCH 260 Physio Psychology chapter 5

View Set

Patent Cooperation Treaty (MPEP 1800)

View Set

Psychology 001-Chapter 7-Learning

View Set

MSII Prep U Ch. 72 Emergency Nursing

View Set

Unit 13 and 14 World War I - Causes, Russia's involvement, the US enters the War, WW I map study

View Set

unit 6 progress check : mcq part a&b

View Set